Вы находитесь на странице: 1из 89

NBME Form 1 Step 2

Section 1:--
1. A 70-year-old woman has had increasin a!dominal pain o"er the past 2 days. She has renal
#ail$re and has !een recei"in peritoneal dialysis #or 1% months& her last treatment was 2 ho$rs
ao. She appears to'ic. (er temperat$re is )* + ,102.2 F-. and !lood press$re is 1/00*0 mm (.
(er a!domen is distended and di##$sely tender to deep palpation with re!o$nd tenderness.
1e$2ocyte co$nt is 1%.0000mm). 3hich o# the #ollowin is the most appropriate ne't step4
A - 5-ray #ilms o# the a!domen
B - +omparison o# a!dominal #l$id amylase with ser$m amylase acti"ity
+ - 6ram7s stain o# a!dominal #l$id
8 - 9ltrasonoraphy o# the a!domen
E - +: scan o# the a!domen and pel"is
2. A ;-year-old irl is !ro$ht to the physician !eca$se o# temperat$res to /0 + ,10/ F-. tachypnea.
and a nonprod$cti"e co$h #or 12 ho$rs. Fo$r days ao she was treated with an oral anti!iotic #or
s$spected pne$mococcal pne$monia. E'amination shows diminished !reath so$nds o"er the lower
riht l$n #ields and d$llness to perc$ssion at the riht costophrenic anle. 3hich o# the #ollowin is
the most li2ely dianosis4
A - Bronchople$ral #ist$la
B - Empyema
+ - 1$n a!scess
8 - <le$rodynia
E - <ne$mothora'
). A /0-year-old man is !ro$ht to the emerency department 1 ho$r a#ter a hih-speed motor
"ehicle collision. =n arri"al. he is awa2e and alert !$t has se"ere pain o"er the stern$m. (is systolic
!lood press$re is %0 mm (. p$lse is %00min. and respirations are 100min. An E+6 shows m$lti#ocal
premat$re "entric$lar contractions !$t no S:-sement chanes. (is <=2 is 100 mm (. A#ter 1 1 o#
lactated >iner7s sol$tion is administered. his <=2 decreases to ?0 mm ( while !reathin / 10min
o# o'yen !y nasal cann$la. <$lmonary capillary wede press$re has increased #rom 1/ mm ( to
2/ mm ( ,N@1A10-. 3hich o# the #ollowin is the most li2ely e'planation #or the patient7s poor
response to #l$id res$scitation4
A - BnadeC$ate administration o# #l$ids
B - Myocardial cont$sion
+ - Myocardial in#arction
8 - <$lmonary cont$sion
E - :ra$matic r$pt$re o# the aorta
/. A 21-year-old A#rican American collee st$dent has had increasin #ati$e o"er the past ) wee2s.
Since an episode o# cystitis treated with trimethoprim-s$l#ametho'aDole ) wee2s ao. he has !een
$na!le to 2eep $p with his physical ed$cation classes. For ? months. he has !een #ollowin a
"eetarian diet that has !een s$per"ised !y st$dent health ser"ices. E'amination shows no
a!normalities. (is hemolo!in le"el is 10 0d1. mean corp$sc$lar "ol$me is %; Em). and
retic$locyte co$nt is 1;F. 3hich o# the #ollowin is the most li2ely ca$se o# anemia in this patient4
A - Anemia secondary to in#ection
B - Anti!iotic therapy
+ - 6astrointestinal !lood loss
8 - Sic2le cell disease
E - Geetarian diet
;. An %7-year-old n$rsin home resident with dementia. AlDheimer7s type. is admitted to the
hospital !eca$se o# proressi"e lethary and decreased appetite #or ) days. She had a #l$-li2e
illness #ollowed !y a deep co$h 1 wee2 ao. ="er the past 10 months. she has !een hospitaliDed
once #or !acterial pne$monia. She is responsi"e only to pain#$l stim$li. (er temperat$re is )%.? +
,101.; F-. !lood press$re is 1100?0 mm (. p$lse is 12)0min and re$lar. and respirations are
2%0min. E'amination shows dry m$co$s mem!ranes. :here is no adenopathy. +rac2les are heard in
the riht l$n !ase. An '-ray #ilm o# the chest shows an in#iltrate at the riht l$n !ase. :he
remainder o# the e'amination shows no a!normalities. 3hich o# the #ollowin is the most li2ely
predisposin #actor #or this patient7s pne$monia4
A - 8ecreased airway elasticity
B - 8ecreased !arore#le'
+ - 8ecreased a re#le'
8 - 8ecreased thyroid #$nction
E - 8iastolic cardiac dys#$nction
F - Bmpaired cardiac response to e'ercise
6 - Bmpaired :-lymphocyte #$nction
( - Bmpaired thirst
B - Bncreased l$n compliance
H - >enal salt wastin
?. A 27-year-old woman comes to the physician !eca$se o# #eelins o# an'iety a!o$t attendin her
10-year hih school re$nion. She has a 2-year history o# pro#o$nd an'iety. palpitations. and
sweatin associated with an $neasiness aro$nd people& she a"oids #amily atherins and "isitin
#riends !eca$se she is a#raid o# !ein em!arrassed. She ac2nowledes that this #ear is
$nreasona!le. She does not $se illicit dr$s !$t says that alcohol ma2es her more com#orta!le
aro$nd people. (er !lood press$re is 1)00*0 mm (. and p$lse is %%0min. =n physical e'amination.
she appears healthy and well no$rished. =ccasional wheeDin is heard o"er the le#t l$n #ield. :he
remainder o# the e'amination shows no a!normalities. =n mental stat$s e'amination. she appears
worried. (er le$2ocyte co$nt is *0000mm) with a normal di##erential. 3hich o# the #ollowin is the
most li2ely dianosis4
A - Alcohol a!$se
B - An'iety disorder d$e to a eneral medical condition
+ - Asthma
8 - 6eneraliDed an'iety disorder
E - <anic disorder with aorapho!ia
F - Social pho!ia
:he response options #or the ne't two items are the same. Io$ will !e reC$ired to select one answer
#or each item in the set.
For each patient with $rinary incontinence. select the most li2ely ca$se.
A - 8etr$sor insta!ility
B - Bnterstitial cystitis
+ - ="er#low incontinence
8 - Stress incontinence
E - 9rethra di"ertic$l$m
F - 9rinary #ist$la
7. A pre"io$sly healthy //-year-old woman. ra"ida /. para /. comes to the physician !eca$se o# a
*-month history o# proressi"e loss o# small amo$nts o# $rine while r$nnin& she now has to wear
an a!sor!ent pad. E'amination shows a second-deree cysto$rethrocele.
For each patient with $rinary incontinence. select the most li2ely ca$se.
A - 8etr$sor insta!ility
B - Bnterstitial cystitis
+ - ="er#low incontinence
8 - Stress incontinence
E - 9rethra di"ertic$l$m
F - 9rinary #ist$la
%. =ne day a#ter an $ncomplicated spontaneo$s "ainal deli"ery. a 2)-year-old woman. ra"ida 1.
para 1. has the onset o# loss o# small amo$nts o# $rine. She recei"ed epid$ral anesthesia d$rin
la!or and deli"ery. E'amination shows an episiotomy witho$t e"idence o# hematoma. She is "oidin
;0 to 7; m1 o# $rine at a time. <ost"oid resid$al "ol$me is )00 m1.
:he response options #or the ne't two items are the same. Io$ will !e reC$ired to select one answer
#or each item in the set.
*. An %2-year-old woman is !ro$ht to the physician !y her randda$hter !eca$se o# a ?-wee2
history o# increasin #oret#$lness. She is a retired schoolteacher and li"es independently. (er
randda$hter is concerned !eca$se on se"eral occasions she has le#t the sto"e on when she went
to !ed. 8$rin con"ersations with her randda$hter. she has di##ic$lty remem!erin past e"ents
and seems $nconcerned a!o$t her memory lapses. :he patient descri!es tro$!le sleepin thro$h
the niht and has had a decreased appetite res$ltin in a /.;-2 ,10-l!- weiht loss o"er the past
month. She has a history o# similar symptoms 2 and ; years ao that were s$ccess#$lly treated with
medication. She appears $n2empt and has poor personal hyiene. (er temperat$re is )7 + ,*%.?
F-. !lood press$re is 110070 mm (. and p$lse is %00min and re$lar. Mental stat$s e'amination
shows psychomotor retardation. a #lat a##ect. impaired a!ility to recall past e"ents. and tro$!le
repeatin three n$m!ers in seC$ence. She is $na!le to recall the names o# recent presidents. (er
ser$m $rea nitroen ,B9N- le"el is 2; m0d1. and ser$m creatinine le"el is 1.7 m0d1.
For each patient with coniti"e impairment. select the most li2ely dianosis.
A - Ac$te stress disorder
B - 8ementia. AlDheimer7s type
+ - 8issociati"e amnesia
8 - 6eneral paresis
E - (ead tra$ma
F - (epatolentic$lar deeneration ,3ilson7s disease-
6 - (BG encephalitis
( - ($ntinton7s disease
B - MaJor depressi"e disorder
H - M$lti-in#arct ,"asc$lar- dementia
K - Niacin de#iciency
1 - Normal-press$re hydrocephal$s
M - <ar2inson7s disease
N - <ic27s disease
= - SchiDophrenia. catatonic type
< - Normal ain
10. A /2-year-old comp$ter science pro#essor is !ro$ht to the physician !y her h$s!and. who
reports insidio$s chanes in his wi#e7s personality and !eha"ior. (e reports that she !elie"es that
aliens ha"e !een spea2in to her and tamperin with their heatin and air-conditionin systems. (e
says that she was $pset when she t$rned /0 years old. and her symptoms ha"e de"eloped since
that time. She was adopted. and her #amily history is $n2nown. <hysical e'amination shows
"ermic$lar mo"ements o# the ton$e and !ilateral writhin motions o# the $pper e'tremities. Mental
stat$s e'amination shows indi##erence to her condition and mild to moderate di##ic$lty with memory
and calc$lations.
11. A ;-month-old !oy is !ro$ht #or a #ollow-$p e'amination. (e was !orn at )7 wee2s7 estation
and has had persistent wheeDin since shortly a#ter !irth despite treatment with ne!$liDed and oral
!ronchodilators and oral corticosteroids. (is diet consists o# )2 o$nces o# iron-#orti#ied cow7s mil2-
!ased #orm$la daily. (e appears well no$rished and happy. =n e'amination. there is moderate relie#
o# wheeDin with e'tension o# the nec2. 3hich o# the #ollowin is the most li2ely mechanism o# this
in#ant7s wheeDin4
A - Alleric reaction to cow7s mil2
B - Aspiration o# a #orein !ody
+ - +ompression o# the airway !y a "asc$lar rin
8 - +onc$rrent $pper respiratory tract in#ection
E - <ersistent immat$rity o# l$ns
12. An 1%-month-old !oy is !ro$ht to the emerency department !eca$se he has not $sed his le#t
arm since he #ell while wal2in and holdin hands with his %-year-old sister 2 ho$rs ao. =n
e'amination. he holds his le#t $pper e'tremity at his side with his #orearm pronated. :here is no
tenderness o# the le#t lower e'tremity. !$t there is restricted mo"ement o# the el!ow. :he
remainder o# the e'amination shows no a!normalities. 3hich o# the #ollowin is the most
appropriate initial step in manaement4
A - <assi"e hypers$pination o# the #orearm
B - Application o# #i$re-o#-% strap
+ - Administration o# analesics and application o# ice
8 - Aspiration o# the el!ow Joint
E - Bn-place splint immo!iliDation o# the el!ow
1). A ?-month-old irl is !ro$ht to the physician #or a ro$tine health maintenance e'amination.
She was !orn with a l$m!osacral myelomeninocele which was s$ccess#$lly repaired at 2 days o#
ae. :he anterior #ontanelle is ? ' % cm and !$lin. and the posterior #ontanelle is ) ' / cm and
!$lin. She has se"ere motor and sensory de#icits in"ol"in !oth lower e'tremities. A head rowth
chart shows the #ollowin "al$es:
Ae
(ead circ$m#erence ,cm-
Birth
)/.2
1 month
)?.?
2 months
)%
/ months
//
? months
/7
3hich o# the #ollowin is the most li2ely ca$se o# increased intracranial press$re4
A - Ac$te cere!ral edema
B - 8ecreased a!sorption o# cere!rospinal #l$id
+ - 8ilation o# cere!ral arteries
8 - Bntracranial mass lesion
E - =!str$ction o# lateral sin$s
F - =!str$ction o# s$perior "ena ca"a
6 - =!str$ction o# cere!rospinal #l$id #low
( - ="erprod$ction o# cere!rospinal #l$id
1/. An 1%-year-old primira"id woman comes #or her initial prenatal "isit at 1? wee2s7 estation.
She is not s$re a!o$t the date o# her last menstr$al period !$t says that the prenancy pro!a!ly
occ$rred immediately a#ter she stopped ta2in oral contracepti"es ; months ao. Maternal ser$m L-
#etoprotein ,MSAF<- le"el is increased to ) m$ltiples o# the median. 3hich o# the #ollowin is the
most appropriate ne't step in manaement4
A - >epeat meas$rement o# MSAF< le"el
B - :riple screenin #or MSAF<. ser$m M-h+6. and ser$m estriol le"els
+ - 9ltrasonoraphy
8 - Amniocentesis #or meas$rement o# L-#etoprotein le"el
E - Amniocentesis #or chromosomal analysis
1;. A 2%-year-old man is !ro$ht to the emerency department !y police !eca$se o# se"ere pain in
the riht lower C$adrant o# the a!domen #or )? ho$rs. (e has !een incarcerated in the co$nty Jail.
and his co$rt hearin is sched$led in 12 ho$rs. (e ta2es no medications. A!dominal e'amination
shows no re!o$nd. +omplete !lood co$nt. li"er tests. and erythrocyte sedimentation rate are within
normal limits. An '-ray #ilm o# the a!domen shows a normal as pattern. :est o# the stool #or occ$lt
!lood is neati"e. 3hich o# the #ollowin is the most li2ely dianosis4
A - AdJ$stment disorder
B - +on"ersion disorder
+ - 8epressi"e disorder not otherwise speci#ied
8 - Factitio$s disorder
E - (ypochondriasis
F - Malinerin
6 - SomatiDation disorder
1?. A ?-year-old irl is !ro$ht to the physician !eca$se o# a /-wee2 history o# headache. #ati$e.
and decreased appetite. 8$rin this period. she has had na$sea and "omitin. At the ae o# /
years. she was dianosed with poststreptococcal lomer$lonephritis. She is at the 1;th percentile
#or heiht and the 10th percentile #or weiht. E'amination shows no a!normalities. (er ser$m $rea
nitroen ,B9N- le"el is ;0 m0d1. 3hich o# the #ollowin is most li2ely to limit proression o# this
patient7s renal #ail$re4
A
- Bncreased potassi$m and sodi$m !icar!onate inta2e
B
- 8ecreased sodi$m and daily calorie inta2e
+
- 1ow-protein diet
8
- Strict #l$id restriction
E
- 8ialysis
17. A pre"io$sly healthy ?2-year-old man comes to the emerency department !eca$se o#
a!dominal pain #or /% ho$rs. (is temperat$re is )%.? + ,101.; F-. !lood press$re is 1)00%0 mm (.
p$lse is 1100min. and respirations are 1;0min. A!dominal e'amination shows di##$se le#t lower
C$adrant tenderness with no peritoneal sins. >ectal e'amination shows no a!normalities& test o#
the stool #or occ$lt !lood is neati"e. (is le$2ocyte co$nt is 1/.7000mm). 3hich o# the #ollowin is
the most appropriate ne't step in dianosis4
A
- Bari$m enema
B
- +: scan o# the a!domen
+
- +olonoscopy
8
- +ystoscopy
E
- E'ploratory laparotomy
1%. A pre"io$sly healthy )-month-old irl is !ro$ht to the emerency department !eca$se o# a )-
day history o# r$ntin and increasin di##ic$lty !reathin. She appears ill. (er temperat$re is )?.7
+ ,*% F-. p$lse is 1?00min. and respirations are 7?0min. E'amination shows r$ntin. nasal #larin.
and mar2ed intercostal retractions. Bronchial !reath so$nds and occasional !ilateral crac2les are
heard on a$sc$ltation. Ser$m st$dies show:
+a2N ;.? m0d1
<hosphor$s 11 m0d1
Al2aline phosphatase 2;0 901
+apillary !lood as analysis on 100F o'yen:
p( 7.)?
<+=2 )% mm (
<=2 /? mm (
An '-ray #ilm o# the chest shows !ilateral. di##$se interstitial in#iltrates and a!sence o# the thymic
shadow. Bronchoal"eolar la"ae is positi"e #or n$mero$s <ne$mocystis carinii. 3hich o# the
#ollowin is the most li2ely mechanism o# these #indins4
A - Adenosine deaminase de#iciency
B - +ons$mption o# complement
+ - 8e#ecti"e opsoniDation
8 - 8estr$ction o# +8/N : lymphocytes
E - 8e"elopmental arrest o# mat$ration o# B lymphocytes
F - 8ysmorphoenesis o# the third and #o$rth pharyneal po$ches
6 - Bmpaired chemota'is
( - Bmpaired phaocytic o'idati"e meta!olism
1*. A /-year-old !oy is !ro$ht to the emerency department 20 min$tes a#ter !ein in"ol"ed in a
motor "ehicle collision. (e was an $nrestrained passener. =n arri"al. his !lood press$re is 110070
mm (. p$lse is 1000min. and respirations are )20min with r$ntin and retractions. E'amination
shows m$ltiple !r$ises o"er the chest. Arterial !lood as analysis while !reathin /0F o'yen
shows:
p( 7.)%
<+=2 )/ mm (
<=2 ?? mm (
An '-ray #ilm o# the chest o!tained / ho$rs later shows di##$se in#iltrates on the riht side. 3hich o#
the #ollowin is the most li2ely dianosis4
A
- Ac$te respiratory distress syndrome
B
- Aspiration pne$monia
+
- Fat em!olism
8
- (emothora'
E
- <$lmonary cont$sion
20. A 72-year-old man comes to the physician !eca$se o# a 7-month history o# le wea2ness and
dry eyes and mo$th. (e also has had a 10./-2 ,2)-l!- weiht loss o"er the past / months despite
no chane in appetite. (e has smo2ed one and a hal# pac2s o# ciarettes daily #or ;0 years. (e
drin2s / oD o# alcohol daily. (e has peptic $lcer disease and emphysema. Medications incl$de
cimetidine. theophylline. and low-dose prednisone. E'amination shows mild ptosis. (e has a !arrel-
shaped chest. Breath so$nds are distant. :here is moderate wea2ness o# the pro'imal m$scles o#
the lower e'tremities. >e#le'es are a!sent. (e has di##ic$lty risin #rom a chair. Sensory
e'amination shows no a!normalities. An '-ray #ilm shows a hyperin#lated chest and a ) ' /-cm
mass in the riht hil$m. (is ne$roloic #indins are most li2ely d$e to a lesion in"ol"in which o# the
#ollowin4
A
- M$scle mem!rane
B
- <arasympathetic ner"o$s system
+
- <eripheral ner"e
8
- <resynaptic ne$rom$sc$lar J$nction
E
- Sympathetic ner"o$s system
21. A /2-year-old man comes to the emerency department !eca$se o# a 2-wee2 history o#
increasinly se"ere headaches and a 2-day history o# na$sea. "omitin. nec2 sti##ness. and
$nsteadiness. (e has type 2 dia!etes mellit$s treated with ly!$ride. (is temperat$re is )%.1 +
,100.; F-. F$nd$scopic e'amination shows !ilateral papilledema. Ne$roloic e'amination shows
mild meninism$s and di##$sely !ris2 deep tendon re#le'es. (e wal2s with a moderately !road-
!ased ait. (e is a!le to recall two o$t o# three o!Jects a#ter ; min$tes and ma2es se"eral errors on
serial se"ens. A +: scan o# the head shows no a!normalities. +ere!rospinal #l$id analysis shows a
l$cose le"el o# 1% m0d1. a protein le"el o# 10% m0d1. and a le$2ocyte co$nt o# ;*0mm) ,1F
semented ne$trophils and **F lymphocytes-& a cryptococcal antien assay is positi"e. 3hich o#
the #ollowin is the most appropriate pharmacotherapy #or this patient4
A
- Acyclo"ir
B
- Amphotericin B
+
- BtraconaDole
8
- <enicillin
E
- Gancomycin
22. A )-year-old irl is !ro$ht to the physician a#ter her mother noted !lood on her $nderpants.
E'amination shows enital condylomata ac$minata in the perineal. peri-introital. la!ial. and anal
areas. Some o# the ped$nc$lated condylomata appear to ha"e ca$sed the !leedin. She has no
"isi!le intra"ainal condylomata or "ainal or anal tears. (er mother has a palmar wart on her hand
!$t no history o# condylomata ac$minata. (er mother has a !oy#riend who does not li"e with them
and who has ne"er !een le#t alone with the irl. :hey li"e with the mother7s 27-year-old !rother
who only !a!y-sits the children when they are asleep. 3hich o# the #ollowin is the most
appropriate ne't step in manaement4
A
- <sychiatric assessment o# the mother
B
- 8NA typin o# the mother7s palmar wart #or papilloma"ir$s
+
- :reatment o# the mother7s palmar wart
8
- Gainal. anal. and throat c$lt$res #or +hlamydia trachomatis and Neisseria onorrhoeae in the
child
E
- 1aser therapy o# the condylomata ac$minata in the child
2). A ;;-year-old man has had cr$shin s$!sternal chest pain on e'ertion o"er the past ? wee2s.
(e had a myocardial in#arction 2 months ao. (e ta2es nitrolycerin as needed and one aspirin
daily. (e has smo2ed two pac2s o# ciarettes daily #or )0 years. E'amination shows normal heart
so$nds and no carotid or #emoral !r$its. :reatment with a M-adreneric !loc2in aent is most li2ely
to impro"e his symptoms d$e to which o# the #ollowin mechanisms4
A
- 8ecreasin diastolic rela'ation
B
- 8ecreasin myocardial contractility
+
- 8ilatin the coronary arteries
8
- <eripheral "asodilation
E
- <re"entin #i!rin and platelet pl$s
2/. A pre"io$sly healthy ;2-year-old woman comes to the physician !eca$se she has had a lare
pimple on her riht hand #or 2 wee2s that has #ailed to heal. She resides in so$theastern 9SA where
she owns a n$rsery and arden shop. E'amination shows a painless red pap$le on the hand with
se"eral nontender s$!c$taneo$s nod$lar lesions a!o"e it. 3hich o# the #ollowin is the most li2ely
dianosis4
A - Blastomycosis
B - +andidiasis
+ - +occidioidomycosis
8 - (istoplasmosis
E - Sporotrichosis
2;. A 2%-year-old woman at 2% wee2s7 estation reports e'cessi"e #atia!ility and dyspnea. (er
!lood press$re is 11%07/ mm (. p$lse is 1100min and re$lar. and l$ns are clear to a$sc$ltation.
:he cardiac ape' is not palpa!le. S1 is lo$d. and there is a sharp so$nd a#ter S2. A low-#reC$ency
diastolic m$rm$r is heard at the ape' that increases in intensity !e#ore S1. 3hich o# the #ollowin is
the most li2ely dianosis4
A
- Aortic re$ritation
B
- E!stein7s anomaly
+
- Mitral re$ritation
8
- Mitral stenosis
E
- :ric$spid re$ritation
2?. A 27-year-old woman comes to the physician !eca$se o# a 2-year history o# intermittent
diarrhea and se"ere crampin a!dominal pain. :he stools are watery. occasionally #o$l-smellin.
and non!loody. She is c$rrently pain-#ree and has not had diarrhea #or 2 days. She also has
intermittent constipation. She has not had #e"er or weiht loss. She ret$rned #rom a trip to Me'ico )
months ao. She had an appendectomy at the ae o# 12 years and a cesarean deli"ery / years ao.
E'amination shows no a!normalities. 3hich o# the #ollowin is the most li2ely dianosis4
A
- Bacterial astroenteritis
B
- +rohn7s disease
+
- Bntermittent small-!owel o!str$ction
8
- Brrita!le !owel syndrome
E
- 1a'ati"e a!$se
27. An asymptomatic 21-year-old woman is #o$nd to ha"e an adne'al mass on pel"ic e'amination.
She $ses oral contracepti"es. A photoraph o# the mass is shown. 3hich o# the #ollowin is the most
li2ely dianosis4
A
- Benin cystic teratoma
B
- +orp$s l$te$m cyst
+
- 8yserminoma
8
- Endometrioma
E
- M$cino$s cystoadenoma
2%. A 1?-year-old irl is !ro$ht to the physician !eca$se o# episodes o# palpitations o"er the past ?
months. :he episodes occ$r when she r$ns or plays !as2et!all. She is otherwise asymptomatic. (er
!lood press$re is 12/0/? mm (. p$lse is 7%0min. and respirations are 1%0min. She weihs ;; 2
,121 l!- and is 1%0 cm ,71 in- tall. (er arm span is 1%% cm ,7/ in-. and the $pper sement to lower
sement ratio is 0.%;. (er #iners appear lon and are hypere'tensi!le. A rade /0?. early diastolic
m$rm$r is heard alon the $pper and middle le#t sternal !order with radiation to the ape'.
<eripheral p$lses are !o$ndin. 3hich o# the #ollowin is the most li2ely ca$se o# these #indins4
A
- Aortic incompetence
B
- Aortic stenosis
+
- Mitral incompetence
8
- Mitral stenosis
E
- <$lmonary incompetence
F
- <$lmonary stenosis
6
- :ric$spid incompetence
(
- :ric$spid stenosis
2*. A pre"io$sly healthy %7-year-old woman comes to the physician !eca$se o# a /-month history
o# "$l"ar itchin. E'amination shows e'coriated areas #rom scratchin and a white. thin "$l"a. :he
la!ia minora are a!sent. and there are small #iss$res at the introit$s. :he remainder o# the
e'amination shows no a!normalities. 3hich o# the #ollowin is the most li2ely dianosis4
A
- Escherichia coli in#ection
B
- 1ichen scleros$s
+
- SC$amo$s cell carcinoma
8
- :richomoniasis
E
- G$l"ar melanoma
F
- G$l"ar "esti!$litis
)0. =ne month a#ter $nderoin an $ne"ent#$l renal transplant #or chronic renal #ail$re secondary
to lomer$lonephritis. a )%-year-old woman is hospitaliDed !eca$se o# increased ser$m $rea
nitroen ,B9N- and creatinine le"els. <rior to transplantation. she had !een recei"in hemodialysis
#or ) years. +$rrent medications incl$de cyclosporine and prednisone. E'amination shows no
a!normalities. ="er the past /% ho$rs. $rine o$tp$t has remained sta!le. Both renal !iopsy and a
radion$clide scan con#irm the dianosis o# ac$te reJection. 3hich o# the #ollowin is the most
e##ecti"e treatment4
A
- Bmmediate discontin$ation o# cyclosporine
B
- Bncreased dosae o# corticosteroids
+
- 8i$resis and al2aliniDation o# the $rine
8
- >enal dialysis #or 1A2 wee2s
E
- :ransplant nephrectomy
)1. An 1%-year-old man comes to the physician 1 wee2 a#ter he had a !lood press$re o# 1/00110
mm ( d$rin a ro$tine precollee e'amination. (is temperat$re is )7.1 + ,*%.7 F-. !lood press$re
is 1/00100 mm (. p$lse is *20min. and respirations are 120min. :he $pper e'tremities appear to
!e more m$sc$lar than the lower e'tremities. >adial p$lses are normal& #emoral. posterior ti!ial.
and dorsalis pedis p$lses are decreased. A rade 20? systolic m$rm$r is heard o"er the precordi$m.
anterior chest. and !ac2. An E+6 shows le#t "entric$lar hypertrophy. 3hich o# the #ollowin is the
most appropriate ne't step in manaement4
A - 1imitin physical acti"ity
B - >epeat !lood press$re meas$rement in 1 month
+ - Bnitiate a low-sodi$m diet and e'ercise proram
8 - <harmacoloic manaement
E - =perati"e treatment
)2. A 77-year-old woman comes to the physician !eca$se o# a 2-day history o# crampin a!dominal
pain and distention accompanied !y na$sea and "omitin. She is otherwise healthy and has no
history o# a!dominal operations. (er temperat$re is )7./ + ,**./ F-. !lood press$re is 1100%? mm
(. p$lse is 1120min. and respirations are 2/0min. +ardiop$lmonary e'amination shows no
a!normalities. E'amination o# the a!domen shows distention and mild di##$se tenderness& !owel
so$nds are hih-pitched. An '-ray #ilm o# the a!domen shows air-#l$id le"els thro$ho$t the small
!owel and air in the li"er& there is no as in the colon or #ree air. 3hich o# the #ollowin is the most
li2ely dianosis4
A
- Adhesi"e small-!owel o!str$ction
B
- +ecal cancer
+
- 6allstone ile$s
8
- Bnt$ss$sception
E
- Mesenteric in#arction
F
- >$pt$red appendicitis
6
- Small !owel lymphoma
)). Fo$r ho$rs a#ter $nderoin a cesarean deli"ery at term #ollowed !y t$!al liation. a )7-year-
old woman. ra"ida 2. para 2. has diDDiness and con#$sion. :he operation was $ncomplicated. and
!lood loss is estimated to !e %00 m1. <atient-controlled epid$ral analesia has !een moderately
e##ecti"e #or pain. (er !lood press$re now is %00/0 mm (. decreased #rom 120072 mm (
intraoperati"ely. and p$lse is 1;20min. increased #rom *?0min intraoperati"ely. Breath so$nds are
decreased !ilaterally. No m$rm$rs are heard. A!dominal e'amination shows distention and
tenderness. Bowel so$nds are a!sent. :he incision is intact with no drainae. She is disoriented to
person. place. and time. (er hematocrit is 2)F& preoperati"e hematocrit was );F. 3hich o# the
#ollowin is the most li2ely ca$se o# the hemodynamic chanes4
A
- Epid$ral-related hypotension
B
- Bns$##icient intraoperati"e #l$id replacement
+
- <ostoperati"e intra-a!dominal hemorrhae
8
- S$pine hypotensi"e syndrome
E
- 9nderestimated intraoperati"e !lood loss
)/. A /2-year-old woman comes to the physician #or e"al$ation o# persistently increased !lood
press$res. At her last two o##ice "isits d$rin the past ) months. her !lood press$re has raned
!etween 1;0A170010;A11; mm (. 8$rin this period. she has had occasional headaches. Bn
addition. she has had an increased $rine o$tp$t o"er the past ? wee2s that she attri!$tes to a diet
hih in sodi$m. She is otherwise healthy and ta2es no medications. (er !lood press$re today is
1?%011; mm (. p$lse is ?%0min. and respirations are 1/0min. F$nd$scopic e'amination shows
mild arterio"eno$s nic2in. :he point o# ma'imal imp$lse is not displaced. :here is no edema.
a!dominal !r$its. or masses. Ser$m st$dies show:
NaN
1// mEC01
+lA
*0 mEC01
KN
2.* mEC01
(+=)A
)2 mEC01
9rea nitroen ,B9N-
20 m0d1
+reatinine
1.2 m0d1
3hich o# the #ollowin is the most li2ely $nderlyin ca$se o# this patient7s hypertension4
A
- A$tonomo$s prod$ction o# aldosterone
B
- +atecholamine-prod$cin t$mor
+
- 8ecreased arterial distensi!ility ca$sed !y atherosclerosis
8
- E'cess prod$ction o# atrial natri$retic peptide
E
- H$'talomer$lar cell hypertrophy and sclerosis
);. A pre"io$sly healthy /-year-old irl is !ro$ht to the physician !eca$se o# #e"er and re#$sal to
wal2 #or 1 day. She appears mildly ill. (er temperat$re is )%.? + ,101.; F-. p$lse is 1200min. and
respirations are 220min. :he riht 2nee is erythemato$s and swollen. She holds her riht 2nee in
#le'ion and resists any attempted mo"ement o# her riht le. She cries when the riht 2nee is
mo"ed. 3hich o# the #ollowin is the most appropriate ne't step in manaement4
A
- Acetaminophen with codeine therapy
B
- Arthrocentesis
+
- Bone marrow aspiration
8
- Bone scan
E
- Bmmo!iliDation and traction
F
- 1yme titer
6
- M>B o# the spine
(
- <hysical therapy
B
- >eass$rance
H
- Ser$m rhe$matoid #actor assay
K
- Systemic anti!iotic therapy
)?. An 1%-year-old man comes #or an e'amination prior to participation in school sports. (e states
that he has had a d$ll ache in the scrot$m since !ein hit in that area d$rin a !as2et!all ame 2
months ao. E'amination shows a 2-cm. hard. nontender mass in the riht testicle. :he mass does
not transill$minate or chane in siDe when the patient is placed in the s$pine position. 3hich o# the
#ollowin is the most li2ely ca$se4
A
- +ystic dilations o# the e##erent d$ct$les
B
- 8ilated pampini#orm "eno$s ple'$s
+
- Fl$id acc$m$lation within the t$nica "ainalis testis
8
- 6erminal cell t$mor
E
- Gasc$lar tra$ma
)7. A 1/-year-old !oy is !ro$ht to the physician !y his parents !eca$se o# a 2-year history o#
increasin academic pro!lems. (is parents say that he has always !een hyperacti"e and
distracti!le. !$t now his academic per#ormance has deteriorated to the point that he is #ailin ninth
rade. (is teachers say that his hyperacti"ity is disr$ptin the classroom. (e weihs ;/ 2 ,120 l!-
and is 1;2 cm ,?0 in- tall. Se'$al de"elopment is :anner stae ;& e'amination shows macro-
orchidism. which was not shown on pre"io$s e'aminations. (e has a hih #orehead and lon.
protr$din ears. (e e'hi!its poor eye contact d$rin the e'amination. <sychoed$cational testin
shows an BO o# 70. 3hich o# the #ollowin is the most li2ely dianosis4
A
- Attention-de#icit0hyperacti"ity disorder
B
- A$tistic disorder
+
- 8own syndrome
8
- Fetal alcohol syndrome
E
- Fraile 5 syndrome
F
- 1esch-Nyhan syndrome
6
- <er"asi"e de"elopmental disorder. not otherwise speci#ied
(
- <rader-3illi syndrome
B
- >ett7s disorder
H
- Semini#ero$s t$!$le dysenesis ,Kline#elter7s syndrome-
)%. A )2-year-old woman comes to the physician !eca$se o# "ainal dischare #or 2 wee2s. She has
!een se'$ally acti"e with one #emale partner #or ; years. She has not !een treated with anti!iotics
o"er the past 2 years. (er last <ap smear was ? years ao when she was se'$ally acti"e with a
male partner. She has not $sed illicit dr$s or alcohol. E'amination shows a rayish "ainal
dischare with a p( reater than /.;. A wet mo$nt preparation o# the "ainal dischare is most
li2ely to show which o# the #ollowin4
A
- B$ddin yeast
B
- +l$e cells
+
- Fernin
8
- 1e$2ocytes in sheets
E
- :richomonas "ainalis
)*. A ;7-year-old woman with !reast cancer comes to the physician !eca$se o# increasin nec2
pain o"er the past ) days. She has #allen #reC$ently !eca$se o# m$scle wea2ness. Gital sins are
within normal limits. E'amination shows hyperre#le'ia o# all e'tremities. :here is tenderness o"er
the cer"ical spine. Ser$m calci$m le"el is 11 m0d1. 5-ray #ilms show metastases to the cer"ical
spine. 3hich o# the #ollowin is the most appropriate ne't step in manaement4
A
- Application o# a so#t cer"ical collar
B
- <hysical therapy
+
- Mithramycin therapy
8
- :amo'i#en therapy
E
- Spinal cord decompression and cer"ical sta!iliDation
/0. A ;7-year-old man comes to the physician !eca$se o# intermittent $rinary incontinence o"er the
past ? months. (e has loss o# small amo$nts o# $rine when he co$hs or sneeDes. (e has not had
pain or !lood with $rination. (e has a 1;-year history o# type 2 dia!etes mellit$s with peripheral
ne$ropathy. retinopathy. and astroparesis. +$rrent medications incl$de metoclopramide and
ly!$ride. (e appears well. >ectal e'amination shows a normal-siDed prostate. Ne$roloic
e'amination shows decreased sensation in a stoc2in-lo"e distri!$tion. Achilles tendon re#le'es are
a!sent !ilaterally. :est o# the stool #or occ$lt !lood is neati"e. 9rinalysis shows 2N protein with no
le$2ocytes or erythrocytes. (is post"oid resid$al "ol$me is ;00 m1. 3hich o# the #ollowin is the
most li2ely mechanism o# this patient7s incontinence4
A
- +entral ner"o$s system disorder
B
- F$nctional incontinence
+
- Bntrinsic sphincter de#iciency
8
- ="er#low incontinence #rom acontractile !ladder
E
- ="er#low incontinence #rom !ladder o$tlet o!str$ction
F
- <el"ic #loor m$scle wea2ness
6
- >etroperitoneal #i!rosis
(
- >etroperitoneal lymphadenopathy
B
- 9rinary tract in#ection
/1. A 27-year-old primira"id woman at )% wee2s7 estation is admitted in la!or. (er prenancy
has !een $ncomplicated. and a ro$tine prenatal "isit 2 days ao showed no a!normalities. =n
admission. #etal heart tones cannot !e heard. 9ltrasonoraphy shows little amniotic #l$id. #etal
edema. and no e"idence o# a #etal heart!eat. A#ter 1 ho$r. she deli"ers a )17;- ,7-l!- still!orn
in#ant& e'amination o# the in#ant shows no o!"io$s a!normalities e'cept #or mild edema. :he
placenta and mem!ranes appear normal. 3hich o# the #ollowin is the most appropriate immediate
co$rse o# action4
A
- Noti#y the hospital lia!ility department
B
- =!tain consent #or #etal oran donation #rom the parents
+
- >ecommend a$topsy o# the in#ant
8
- :ell the mother not to worry since she can et prenant aain
E
- :ell the parents that there is a 1 in / chance o# rec$rrence in #$t$re prenancies
/2. A pre"io$sly healthy 1?-year-old hih school wrestler comes to the physician !eca$se o# a rash
on his #orearms and the !ac2 o# his les #or 1 wee2. (e is alleric to pollen and d$st. E'amination
shows patches o# erythema with mild licheni#ication o"er the antec$!ital and popliteal #ossae. :here
are cl$sters o# pain#$l $m!ilicated "esicles at sites o# acti"e s2in in#lammation. 3hich o# the
#ollowin is the most li2ely dianosis4
A
- EcDema herpetic$m
B
- (erpes Doster
+
- Keratosis pilaris
8
- 1ichen plan$s
E
- <ityriasis rosea
/). A ;-wee2-old !oy is !ro$ht to the physician !eca$se o# "omitin #or ) days. Switchin #rom a
cow7s mil2-!ased #orm$la to a soy-!ased #orm$la and one !ottle o# an electrolyte sol$tion has not
decreased his "omitin. (is mother says that there is no yellow color to the "omit$s. !$t it is
#orce#$l and occ$rs immediately a#ter he has had 1 to 2 o$nces o# liC$id. (e appears to "omit more
liC$id than he dran2. (e has one m$stard-colored seedy stool daily. E'amination shows no
a!normalities. 3hich o# the #ollowin is the most li2ely e'planation #or his "omitin4
A
- 8$odenal atresia
B
- 6astroesophaeal re#l$'
+
- (ypertrophic pyloric stenosis
8
- 1actose intolerance
E
- <rotein mala!sorption
F
- >ota"ir$s in#ection
//. A 2%-year-old woman is hospitaliDed a#ter ta2in a massi"e o"erdose o# acetaminophen ta!lets
in a s$icide attempt. She has type 1 dia!etes mellit$s and maJor depressi"e disorder re#ractory to
tricyclic antidepressant therapy. 8espite appropriate therapy. she de"elops rapidly proressi"e
hepatic #ail$re and !ecomes proressi"ely encephalopathic. =n the ?th day o# hospitaliDation. she is
comatose. A +: scan o# the !rain shows mild di##$se swellin. An appropriately crossmatched. siDe-
appropriate donor li"er is a"aila!le. 3hich o# the #ollowin is the most appropriate co$rse o# action
reardin transplantation4
A
- 8o not proceed with the transplantation !eca$se dia!etes mellit$s is a contraindication
B
- 8o not proceed with the transplantation !eca$se hepatic #$nction is li2ely to ret$rn o"er the ne't
wee2
+
- 8o not proceed with the transplantation !eca$se maJor depressi"e disorder places the patient at
ris2 #or another s$icide attempt
8
- 8o not proceed with the transplantation !eca$se the onset o# encephalopathy and +: #indins
s$est !acterial meninitis
E
- <roceed with the transplantation
/;. A )-year-old !oy who is (BG positi"e is !ro$ht #or a ro$tine e'amination. (is diet is appropriate
#or ae. (is medications incl$de three antiretro"iral dr$s and trimethoprim-s$l#ametho'aDole #or
<ne$mocystis carinii prophyla'is. 1a!oratory st$dies show:
(emolo!in %.? 0d1
Mean corp$sc$lar hemolo!in )% p0cell
Mean corp$sc$lar hemolo!in concentration )0F (!0cell
Mean corp$sc$lar "ol$me 101 Em)
1e$2ocyte co$nt ;?000mm)
Semented ne$trophils ?0F ,many hypersemented-
Bands )F
1ymphocytes )7F
>ed cell distri!$tion width 21F ,N@10A1?-
3hich o# the #ollowin is most li2ely to ha"e pre"ented this patient7s anemia4
A
- Folic acid s$pplementation
B
- Bron s$pplementation
+
- :hyroid s$pplementation
8
- Gitamin B12 ,cyanoco!alamin- s$pplementation
E
- Monthly intra"eno$s imm$ne lo!$lin therapy
/?. A 20-year-old man is !ro$ht to the emerency department on a s$mmer day 20 min$tes a#ter
de"elopin headache. na$sea. and $nsteady ait while r$nnin the last 2 miles o# a marathon. =n
arri"al. he is con#$sed and disoriented. (is temperat$re is /0 + ,10/ F-. !lood press$re is 1000?0
mm (. and p$lse is 1;;0min. (is s2in is warm and dry. Ne$roloic e'amination shows no #ocal
#indins. 3hich o# the #ollowin is the most li2ely mechanism o# this patient7s condition4
A
- 8epletion o# total !ody potassi$m
B
- 8epletion o# total !ody sodi$m
+
- (ih-o$tp$t cardiac #ail$re
8
- BnadeC$ate dissipation o# !ody heat
E
- >elease o# creatine 2inase #rom m$scle cells
PPPPPPPPPPPPPPPPPPPPPPPPPPPPPPPPPPPPPPPPPPPPPPPPPPPPPPPPPPPPPPPPPPPPPPPPPPPPPPP
PPP
Form 1--Section 2:--
1. A 77-year-old woman is !ro$ht to the physician !y her son #or a ro$tine health maintenance
e'amination. She says that she #eels well. (er son reports that 1 month ao. she ot lost while
dri"in home #rom the local s$permar2et. :wo wee2s ao. she #orot to t$rn o## the sto"e a#ter
coo2in dinner. She has !een wearin !ilateral hearin aids since a$diometry 2 years ao showed
!ilateral hih-#reC$ency hearin loss. (er "is$al ac$ity corrected with lasses is 2002; in !oth eyes.
Ne$roloic e'amination shows mild #ine tremors o# the hands when the arms are o$tstretched& the
tremor is not present at rest. M$scle strenth is ;0; in all e'tremities. 8eep tendon re#le'es are
decreased at the an2les and 2N elsewhere. (er ait is normal. Sensation to "i!ration is mildly
decreased o"er the toes. =n mental stat$s e'amination. she is awa2e. alert. and con"ersant. (er
lan$ae #$nction is normal. She is oriented to person. place. and time and recalls one o$t o# three
o!Jects a#ter 10 min$tes. 3hich o# the #ollowin #indins in this patient warrants #$rther e"al$ation4
A
- 8ecreased deep tendon re#le'es at the an2les
B
- 8ecreased sensation to "i!ration o"er the toes
+
- (ih-#reC$ency hearin loss
8
- Memory loss
E
- :remor o# the o$tstretched hands
2. A 10-year-old irl is !ro$ht to the emerency department !eca$se o# di##$se. achin a!dominal
pain. na$sea. and rec$rrent "omitin o"er the past ; ho$rs. She has an %-year history o# type 1
dia!etes mellit$s treated with 20 9 o# N<( and ? 9 o# re$lar ins$lin in the mornin and 1/ 9 o#
N<( and ; 9 o# re$lar ins$lin in the e"enin. She appears letharic !$t is easily aro$sa!le. :here is
an o!"io$s odor o# 2etones on her !reath. (er !lood press$re is 100070 mm (. p$lse is *;0min.
and respirations are 200min and deep. Ser$m st$dies show:
NaN 1/2 mEC01
KN ;.) mEC01
(+=)A ? mEC01
6l$cose 710 m0d1
3hich o# the #ollowin la!oratory #indins is most li2ely to !e increased4
A - Arterial p(
B - Ser$m +-peptide le"el
+ - Ser$m manesi$m le"el
8 - Ser$m osmolality
E - Ser$m phosphor$s le"el
). A healthy 2/-year-old woman comes #or a ro$tine health maintenance e'amination. Menses occ$r
at re$lar 2%-day inter"als and last ; to ? days. (er last menstr$al period was ) wee2s ao. She
ta2es no medications. Biman$al e'amination shows a ;-cm. mildly tender le#t adne'a. A prenancy
test is neati"e. 3hich o# the #ollowin is the most appropriate ne't step in manaement4
A
- >epeat e'amination in 2 wee2s
B
- Meas$rement o# ser$m +A 12; le"el
+
- Meas$rement o# ser$m L-#etoprotein le"el
8
- +: scan o# the pel"is
E
- 8ianostic laparoscopy
/. A 2/-year-old primira"id woman at 1% wee2s7 estation comes #or a ro$tine prenatal "isit. She
has had increased !owel mo"ements o"er the past * wee2s& the stools are sometimes co"ered with
m$c$s and !lood. 9se o# o"er-the-co$nter antidiarrheal dr$s has not relie"ed her symptoms.
<renancy has !een otherwise $ncomplicated. She has ne"er tra"eled o$tside the 9SA. E'amination
shows erythemato$s. tender nod$les o"er the anterior s$r#ace o# !oth lower e'tremities& some o#
the nod$les ha"e a "iolaceo$s h$e. :he $ter$s is consistent in siDe with an 1%-wee2 estation.
>ectal e'amination shows no hemorrhoids or #iss$res. Fetal heart tones are a$di!le !y 8oppler.
3hich o# the #ollowin is the most li2ely dianosis4
A
- Ame!iasis
B
- 8i"ertic$litis
+
- (yperperistaltic diarrhea
8
- Bn#lammatory !owel disease
E
- Giral astroenteritis
;. A )2-year-old woman at )% wee2s7 estation comes #or a ro$tine prenatal "isit. 8$rin ro$tine
screenin at 2% wee2s7 estation. she tested positi"e #or hepatitis B s$r#ace antien. (er prenancy
has !een otherwise $ncomplicated. E'amination shows a $ter$s consistent in siDe with a )%-wee2
estation. 3hich o# the #ollowin meas$res is most li2ely to decrease the ris2 #or hepatitis B
in#ection in her new!orn4
A
- >ecommendation o# !ottle-#eedin rather than !reast-#eedin
B
- Maternal administration o# hepatitis B imm$ne lo!$lin ,(BB6- now
+
- Neonatal administration o# (BB6 a#ter deli"ery and hepatitis B "accine at ) months o# ae
8
- Neonatal administration o# (BB6 and hepatitis B "accine immediately a#ter deli"ery
E
- +esarean deli"ery
?. A )7-year-old woman comes to the physician !eca$se o# proressi"e shortness o# !reath o"er the
past ; years& she now has #ati$e and shortness o# !reath with mild e'ertion. She has a history o#
mitral stenosis secondary to rhe$matic #e"er at the ae o# 1; years. She was asymptomatic $ntil ;
years ao when she de"eloped se"ere shortness o# !reath d$rin prenancy. She was treated with
di$retics. low-sodi$m diet. and !ed rest. and she was a!le to deli"er the !a!y at term. (er only
medication is hydrochlorothiaDide. (er temperat$re is )7 + ,*%.? F-. !lood press$re is 1100%0 mm
(. p$lse is 1000min and re$lar. and respirations are 2?0min. +ardiac e'amination shows an
o!"io$s openin snap in S2. A rade )0?. late diastolic m$rm$r is heard at the ape'. A riht
"entric$lar li#t is palpated alon the le#t sternal !order. 3hich o# the #ollowin is most li2ely
increased in this patient4
A
- Blood #low to the lower l$n #ields
B
- 8iastolic #illin time
+
- 1e#t-to-riht sh$nt o# !lood
8
- 1e#t "entric$lar end-diastolic press$re
E
- <$lmonary artery press$re
7. A ;-year-old irl with "entric$lar septal de#ect is sched$led #or tonsillectomy in 2 wee2s. She has
no 2nown dr$ alleries. (er temperat$re is )7 + ,*%.? F-. E'amination shows no a!normalities.
3hich o# the #ollowin is the most appropriate prophyla'is prior to tonsillectomy4
A
- Amo'icillin
B
- +ipro#lo'acin
+
- >i#ampin
8
- :etracycline
E
- :rimethoprim-s$l#ametho'aDole
F
- No prophyla'is indicated
%. An %-year-old irl with type 1 dia!etes mellit$s is !ro$ht to the emerency department 10
min$tes a#ter !ein in"ol"ed in a motor "ehicle collision. She was in the !ac2 seat o# a small
a$tomo!ile that was rear-ended. Bnitially. she was alert d$rin transport and reported !ilateral thih
pain. !$t then she stopped tal2in. closed her eyes. and !ecame $nresponsi"e to "oice& on arri"al.
she responds to no'io$s stim$li with !rie# rimaces and no withdrawal. (er !lood press$re is
/00palpa!le mm (. p$lse is 1/%0min. and respirations are 2%0min. Air entry is symmetric. :he
p$pils are eC$al and react to liht. No cardiac m$rm$r is heard. :he a!domen is so#t. :here is
swellin o# the $pper portions o# !oth thihs. (er hematocrit is )7F. 3hich o# the #ollowin is the
most appropriate ne't step in manaement4
A
- Meas$rement o# arterial !lood ases
B
- 5-ray #ilm o# the chest
+
- +: scan o# the head
8
- Administration o# ;0F de'trose in water
E
- Bn#$sion o# 0.*F saline
*. A pre"io$sly healthy 1?-year-old !oy is !ro$ht to the emerency department 20 min$tes a#ter
an episode o# le#t arm sha2in that lasted appro'imately ) min$tes. ="er the past 2 days. he has
had #e"er and emotional la!ility. =n arri"al. his temperat$re is )%.* + ,102 F-. (e is somnolent and
disoriented to person. place. and time. (e responds poorly to pain. Ne$roloic e'amination shows
no other a!normalities. 1a!oratory st$dies show:
(ematocrit )/F
1e$2ocyte co$nt ?0000mm)
Semented ne$trophils ;0F
1ymphocytes ;0F
<latelet co$nt 2%0.0000mm)
Analysis o# cere!rospinal #l$id shows:
1e$2ocyte co$nt 1200mm)
Semented ne$trophils 20F
1ymphocytes %0F
Erythrocyte co$nt )000mm)
6l$cose ?0 m0d1
<rotein /00 m0d1
3hich o# the #ollowin is the most li2ely ca$se o# this patient7s ne$roloic #indins4
A
- Bacterial in#ection
B
- +onenital mal#ormation
+
- F$nal in#ection
8
- (emorrhae
E
- Bmm$ne-mediated demyelination
F
- <arasitic in#ection
6
- Giral in#ection
10. A 72-year-old man with hypertension has had increasinly se"ere !ac2 pain o"er the past 2
months. (e had a myocardial in#arction / years ao. (e has mar2ed tenderness o"er :11. :12. 11.
and 12. An '-ray #ilm o# the l$m!osacral spine shows osteo!lastic lesions in these "erte!rae. 3hich
o# the #ollowin is the most li2ely dianosis4
A
- A!dominal ane$rysm
B
- Fi!rosarcoma
+
- Metastatic prostate carcinoma
8
- M$ltiple myeloma
E
- =steosarcoma
11. A ?/-year-old woman has moderately se"ere postoperati"e pain 1 day a#ter a total a!dominal
hysterectomy and !ilateral salpino-oophorectomy. 3hich o# the #ollowin is the most appropriate
analesic pharmacotherapy4
A
- =ral aspirin-codeine compo$nd
B
- =ral diaDepam
+
- =ral i!$pro#en
8
- Bntermittent intra"eno$s nalo'one
E
- <atient-controlled intra"eno$s morphine
F
- :ransc$taneo$s administration o# #entanyl
12. A )7-year-old woman comes to the physician !eca$se o# a 1-day history o# thro!!in #acial
pain. She descri!es the pain as 7 o$t o# 10 in intensity. ="er the past * days. she has had nasal
conestion. p$r$lent nasal dischare. sore throat. and a nonprod$cti"e co$h. She does not smo2e.
(er h$s!and and children ha"e had no recent illness. (er temperat$re is )%.; + ,101.) F-.
E'amination shows conested nasal m$cosa and p$r$lent dischare on the le#t. :here is tenderness
to palpation o"er the le#t chee2 and no transill$mination o"er the le#t ma'illary sin$s. :he tympanic
mem!ranes are normal. and there is no erythema o# the throat. E'amination shows no cer"ical
adenopathy. :he l$ns are clear to a$sc$ltation. 3hich o# the #ollowin is the most li2ely ca$sal
oranism4
A
- (aemophil$s in#l$enDae type !
B
- Mora'ella catarrhalis
+
- Staphylococc$s a$re$s
8
- Streptococc$s pne$moniae
E
- Streptococc$s pyoenes ,ro$p A-
1). A /2-year-old woman comes to the physician #or an ann$al pel"ic e'amination and <ap smear.
="er the past year. she has had increasin #ati$e and di##ic$lty sleepin. She has two children who
!oth attend collee. She is c$rrently loo2in #or part-time wor2 o$tside the home. (er h$s!and has
!een !$sy in a new start-$p !$siness. E'amination shows no a!normalities. 1a!oratory st$dies
show:
(emolo!in 1; 0d1
Mean corp$sc$lar "ol$me *; Em)
1e$2ocyte co$nt ?0000mm) with a normal di##erential
Ser$m
NaN 1/; mEC01
+lA 102 mEC01
KN /.; mEC01
(+=)A 2; mEC01
9rea nitroen ,B9N- 1% m0d1
+reatinine 1.0 m0d1
Al2aline phosphatase 70 901
Aspartate aminotrans#erase
,AS:. 6=:- 22 901
Alanine aminotrans#erase
,A1:. 6<:- 1* 901
Q-6l$tamyltrans#erase
,66:- %) 901
,N@;A;0 901-
3hich o# the #ollowin is the most li2ely e'planation #or this patient7s la!oratory a!normalities4
A
- Acetaminophen
B
- Alcohol
+
- 8iphenhydramine
8
- Estroen e##ect
E
- B!$pro#en
1/. Fi"e wee2s a#ter "ainal deli"ery o# a healthy #$ll-term new!orn. a 22-year-old woman. ra"ida
1. para 1. is !ro$ht to the physician !y her mother !eca$se o# depressed mood #or 2 wee2s. (er
mother is concerned that her da$hter is not a!le to ta2e care o# her in#ant. <hysical e'amination
shows no a!normalities. She is C$iet and tear#$l and does not enae in con"ersation easily. She
states that she li"es alone with her in#ant and has had tho$hts o# s$icide and in#anticide. 3hich o#
the #ollowin is the most appropriate ne't step in manaement4
A
- >eass$rance
B
- 1on-term o$tpatient co$nselin
+
- Antipsychotic therapy
8
- Selecti"e serotonin re$pta2e inhi!itor therapy
E
- Admission to the hospital #or treatment
1;. A 72-year-old man comes #or a ro$tine #ollow-$p e'amination. (e has chronic o!str$cti"e
p$lmonary disease treated with M-adreneric aonists and ipratropi$m !y metered-dose inhaler and
mild arterial ins$##iciency o# the lower e'tremities treated with aspirin. (is !lood press$re is 1?00?0
mm (. p$lse is 700min. and respirations are 120min. F$nd$scopic e'amination shows
arterio"eno$s nic2in. <edal p$lses are decreased !ilaterally. 3hich o# the #ollowin
antihypertensi"e dr$s is most li2ely to ca$se ad"erse e##ects in this patient4
A
- L2-Adreneric aonist
B
- L-Adreneric !loc2in aent
+
- M-Adreneric !loc2in aent
8
- Aniotensin-con"ertin enDyme ,A+E- inhi!itor
E
- +alci$m-channel !loc2in aent
F
- 1oop di$retic
6
- :hiaDide di$retic
(
- Gasodilator
:he response options #or the ne't two items are the same. Io$ will !e reC$ired to select one answer
#or each item in the set.
For each patient with loss o# conscio$sness. select the most li2ely dianosis.
A
- Aortic stenosis
B
- +arotid sin$s hypersensiti"ity
+
- +on"ersion reaction
8
- (ypertrophic o!str$cti"e cardiomyopathy
E
- (ypolycemia
F
- Mitral "al"e prolapse
6
- =rthostatic hypotension
(
- <$lmonary em!ol$s
B
- SeiD$re
H
- Gaso"aal syncope
K
- Gerte!ro!asilar ins$##iciency
1?. A 1;-year-old !oy is !ro$ht to the emerency department )0 min$tes a#ter a 2-min$te episode
o# loss o# conscio$sness a#ter completin a /00-meter race. =n awa2enin. he says that he #eels
#ine e'cept #or shortness o# !reath. (e weihs %2 2 ,1%0 l!- and is 1*1 cm ,7; in- tall. (is !lood
press$re is 110070 mm (. p$lse is 700min and re$lar. and respirations are 1;0min. :he l$ns are
clear to a$sc$ltation. A rade 20? systolic m$rm$r is heard at the le#t sternal !order with minimal
radiation to the nec2& the m$rm$r !ecomes lo$der when he stands.
For each patient with loss o# conscio$sness. select the most li2ely dianosis.
A
- Aortic stenosis
B
- +arotid sin$s hypersensiti"ity
+
- +on"ersion reaction
8
- (ypertrophic o!str$cti"e cardiomyopathy
E
- (ypolycemia
F
- Mitral "al"e prolapse
6
- =rthostatic hypotension
(
- <$lmonary em!ol$s
B
- SeiD$re
H
- Gaso"aal syncope
K
- Gerte!ro!asilar ins$##iciency
17. A ?2-year-old woman is !ro$ht to the emerency department 1 ho$r a#ter a 1-min$te episode
o# loss o# conscio$sness& her symptoms !ean when she stood $p a#ter she passed a dar2. watery
stool. She has had diarrhea and dar2 stools #or 2 days. She has !een recei"in war#arin therapy #or
deep "eno$s throm!osis #or 2 wee2s. =n arri"al. her !lood press$re is %20?0 mm (. and p$lse is
1;00min and re$lar. She is $na!le to stand. :he l$ns are clear to a$sc$ltation. A rade 20?
systolic m$rm$r is heard at the second riht intercostal space with no radiation. E'amination shows
a so#t. nontender a!domen. :here is 1N edema o# the riht lower e'tremity with no tenderness.
:est o# the stool #or occ$lt !lood is positi"e.
1%. A ;0-year-old woman has had proressi"e dyspnea o"er the past 2 wee2s and constant. sharp
chest pain #or / days. :he pain is localiDed to the center o# the chest and is worse while s$pine. She
$nderwent a riht. modi#ied radical mastectomy and adJ$"ant chemotherapy #or !reast cancer )
years ao. She has a history o# hypothyroidism treated with thyroid replacement therapy. She has
smo2ed one pac2 o# ciarettes daily #or )0 years and drin2s two o$nces o# alcohol daily. She is
dyspneic and diaphoretic. (er temperat$re is )7.2 + ,** F-. !lood press$re is *0070 mm ( with a
p$ls$s parado'$s o# 20 mm (. p$lse is 1100min. and respirations are 2%0min. E'amination shows
J$$lar "eno$s distention to the anle o# the mandi!le. :he li"er span is 1/ cm with / cm o# shi#tin
a!dominal d$llness. Arterial !lood as analysis on room air shows a p( o# 7.;0. <+=2 o# )0 mm (.
and <=2 o# 70 mm (. An '-ray #ilm o# the chest shows an enlared cardiac silho$ette with a
lo!$lar con#i$ration. An E+6 shows sin$s tachycardia with nonspeci#ic S:-sement chanes
di##$sely. 3hich o# the #ollowin is the most appropriate ne't step in manaement4
A - Echocardioraphy
B - +: scan o# the a!domen
+ - Gentilation-per#$sion l$n scans
8 - Bronchoscopy
E - <aracentesis
1*. A )-year-old !oy is !ro$ht to the physician !eca$se o# a 7-day history o# #e"er and a pain#$l
swollen lymph node in his roin. :his is his si'th episode o# lymph node swellin& the pre"io$s
episodes resol"ed a#ter drainae and proloned anti!iotic therapy. (e also had pne$monia at the
ae o# 12 months that reC$ired chest t$!e placement #or drainae. A maternal $ncle died d$rin
childhood o# rec$rrent in#ections. :he patient is at the ;th percentile #or heiht and weiht. (is
temperat$re is )%.; + ,101.) F-. E'amination shows a warm. tender. erythemato$s lymph node in
the riht in$inal area. :here are se"eral healed incisions o"er the in$inal area and nec2 #rom old
drainae sites. 1a!oratory st$dies show:
(ematocrit );F
1e$2ocyte co$nt 17.0000mm)
Semented ne$trophils ?;F
Bands 10F
1ymphocytes 2;F
<latelet co$nt );0.0000mm)
A 6ram7s stain o# the lymph node aspirate shows n$mero$s semented ne$trophils #illed with
!acteria& c$lt$res row Staphylococc$s a$re$s. 3hich o# the #ollowin is the most li2ely mechanism
#or these #indins4
A
- Adenosine deaminase de#iciency
B
- +ons$mption o# complement
+
- 8e#ecti"e opsoniDation
8
- 8estr$ction o# +8/N : lymphocytes
E
- 8e"elopmental arrest o# mat$ration o# B lymphocytes
F
- 8ysmorphoenesis o# the third and #o$rth pharyneal po$ches
6
- Bmpaired chemota'is
(
- Bmpaired phaocytic o'idati"e meta!olism
20. A ?7-year-old woman has !een int$!ated #or 1 wee2 a#ter $nderoin a le#t lo!ectomy #or l$n
cancer. She has chronic o!str$cti"e p$lmonary disease. (er preoperati"e #$nctional "ital capacity
was /0F o# predicted. She is awa2e and alert. (er !lood press$re is 1)007; mm (. and p$lse is
720min. :he "entilator settins are a synchroniDed intermittent mandatory "entilation o# %0min.
FB=2 o# /0F. and positi"e-end e'piratory press$re o# ; cm (2=. Arterial !lood as analysis shows:
p( 7./2
<+=2 /7 mm (
<=2 *0 mm (
=2 sat$ration *?F
3hich o# the #ollowin is the most appropriate ne't step in manaement4
A
- Anti!iotic therapy
B
- Bronchodilator therapy
+
- +hest physiotherapy
8
- 8ecrease inotropes
E
- 8i$retic therapy
F
- Fi!eroptic !ronchoscopy
6
- (eparin therapy
(
- Bncenti"e spirometry
B
- Bncrease FB=2
H
- Bncrease inotropes
K
- Bncrease respiratory rate
1
- <lacement o# thoracostomy t$!e
M
- :racheostomy
N
- 3ean #rom the "entilator
21. A ?7-year-old woman is !ro$ht to the emerency department !eca$se o# se"ere chest pain /
ho$rs a#ter $nderoin o$tpatient endoscopy and dilatation o# an esophaeal strict$re ca$sed !y
re#l$'. At dischare. she reported no chest pain. :hree ho$rs later. she "omited a small amo$nt o#
!lood and had se"ere pain. She is pale. (er temperat$re is )% + ,100./ F-. !lood press$re is 1/00%;
mm (. p$lse is 12;0min. and respirations are 220min. E'amination shows crepit$s in the nec2 and
moderate epiastric tenderness. :he l$ns are clear to a$sc$ltation. and !reath so$nds are eC$al
!ilaterally. >ectal e'amination shows no masses& test o# the stool #or occ$lt !lood is positi"e. 3hich
o# the #ollowin is the most li2ely ca$se o# these symptoms4
A
- Bleedin #rom erosi"e esophaitis
B
- Esophaeal per#oration
+
- Mallory-3eiss syndrome
8
- Myocardial in#arction
E
- <er#orated astric $lcer
22. An %7-year-old woman is !ro$ht to the physician !y her son !eca$se o# proressi"e memory
loss o"er the past 2 years. (er son says that she repeats hersel# #reC$ently and has !een #orettin
to ta2e her ro$tine medications. She ta2es hydrochlorothiaDide #or mild systolic hypertension and
le"othyro'ine #or hypothyroidism. She had "$l"ar cancer 10 years ao treated with wide e'cision.
(er !lood press$re is 1)%07% mm (. <hysical e'amination is within normal limits #or her ae. Mini-
Mental State E'amination score is 2)0)0. 1a!oratory st$dies. incl$din ser$m "itamin B12
,cyanoco!alamin-. thyro'ine ,:/-. and thyroid-stim$latin hormone le"els. are within normal limits.
A +: scan o# the head shows mild "ol$me loss. 3hich o# the #ollowin is the most appropriate
pharmacotherapy4
A
- M-Adreneric aonist
B
- +holinesterase inhi!itor
+
- 8opamine aonist
8
- <rednisone
E
- Selecti"e serotonin re$pta2e inhi!itor
2).
A new!orn is in se"ere respiratory distress immediately #ollowin deli"ery. She was !orn at );
wee2s7 estation to a );-year-old woman. ra"ida 2. para 1. a!orta 1. who did not recei"e prenatal
care. :he new!orn7s p$lse is ?00min. and respirations are irre$lar and la!ored. E'amination shows
pallor with perioral cyanosis. anasarca. hepatosplenomealy. and scattered petechiae. +ord !lood
hemolo!in is / 0d1. and retic$locyte co$nt is 1%F. A direct antilo!$lin ,+oom!s7- test is positi"e.
3hich o# the #ollowin sets o# !lood ro$ps is most li2ely in the mother and her new!orn4
Mother New!orn
A
-
A. >h-positi"e =. >h-positi"e
B
-
A. >h-positi"e =. >h-neati"e
+
-
A. >h-neati"e =. >h-neati"e
8
-
=. >h-positi"e =. >h-neati"e
E
-
=. >h-neati"e =. >h-positi"e
2/. A#ter an $ncomplicated laparoscopic cholecystectomy. a ?2-year-old man has not had any $rine
o$tp$t since the Foley catheter was remo"ed 12 ho$rs ao. 8$rin the ho$r !e#ore the operation.
the /0 min$tes o# operatin room time. and the 2 ho$rs in the reco"ery room. his #l$id inp$t was
2.; 1 and $rine o$tp$t was 1 1. Since that time. he has !een recei"in intra"eno$s ;F de'trose in
water with 0./;F saline and morphine. (e is awa2e and alert and has a moderate amo$nt o#
a!dominal pain. <reoperati"e ser$m st$dies showed:
NaN 1)7 mEC01
KN /.2 mEC01
9rea nitroen ,B9N- 1% m0d1
+reatinine 1.2 m0d1
=ne ho$r a#ter recei"in an intra"eno$s !ol$s o# 0.*F saline. the patient does not prod$ce any
$rine. 3hich o# the #ollowin is the most appropriate ne't step in manaement4
A
- Bncrease in the dose o# morphine
B
- Bntra"eno$s administration o# an additional !ol$s o# 0.*F saline
+
- Bntra"eno$s administration o# do'aDosin
8
- Bntra"eno$s administration o# #$rosemide
E
- >einsertion o# a Foley catheter
2;. A healthy ;;-year-old man comes #or an initial health maintenance e'amination. (is last "isit to
a physician was o"er 10 years ao. (e does not smo2e and drin2s only on social occasions.
E'amination shows no a!normalities. 3hich o# the #ollowin imm$niDations sho$ld !e administered4
A
- (epatitis A "accine
B
- Bn#l$enDa "ir$s "accine
+
- Measles-m$mps-r$!ella "accine
8
- <ne$mococcal "accine
E
- 8iphtheria-tetan$s to'oid
2?. A 1*-year-old man comes to the physician !eca$se o# #reC$ent nose!leeds o"er the past )
wee2s. (e has !ipolar disorder c$rrently well controlled with lithi$m car!onate. !$propion. and
"alproic acid. <hysical e'amination shows no a!normalities e'cept #or dried !lood in the nares.
Mental stat$s e'amination shows an an'io$s mood and sliht motor restlessness. Ser$m st$dies
show a lithi$m car!onate le"el o# 1.) mEC01 ,therape$tic rane@0.?A1.2-. and "alproic acid le"el o#
77 E0m1 ,therape$tic rane@/0A100-. 3hich o# the #ollowin is the most appropriate ne't step in
manaement4
A
- Meas$rement o# ser$m aspartate aminotrans#erase ,AS:. 6=:- acti"ity
B
- Meas$rement o# ser$m !$propion le"el
+
- <latelet co$nt
8
- 8iscontin$ation o# lithi$m car!onate therapy
E
- 8iscontin$ation o# "alproic acid therapy
27. An asymptomatic )2-year-old man comes #or a ro$tine health maintenance e'amination. (e has
a 10-year history o# #reC$ent sin$s and p$lmonary in#ections. (e had an anaphylactic reaction to a
!lood trans#$sion #ollowin a motor "ehicle collision ) years ao. (is temperat$re is )7 + ,*%.? F-.
E'amination shows mild erythema in the posterior pharyn'. :he l$ns are clear to a$sc$ltation. A
complete !lood co$nt and ser$m protein electrophoresis are within normal limits. 3hich o# the
#ollowin is the most li2ely ca$se o# the #reC$ent in#ections4
A
- +oloniDation with Streptococc$s pne$moniae
B
- +ommon "aria!le imm$node#iciency
+
- (BG in#ection
8
- Selecti"e BA de#iciency
E
- 5-lin2ed aammalo!$linemia
2%.
A )7-year-old man is !ro$ht to the emerency department ? ho$rs a#ter the onset o# constant.
increasinly se"ere a!dominal pain and na$sea. (is symptoms awo2e him #rom sleep. and he has
"omited once since that time. (e has no history o# similar symptoms. and he does not ta2e any
medications or $se alcohol or illicit dr$s. Family history is noncontri!$tory. (e is in ac$te distress
and lyin in the #etal position. Any mo"ement e'acer!ates the pain. (is temperat$re is )7.% + ,100
F-. !lood press$re is 10%0?% mm (. p$lse is 1120min. and respirations are 2/0min. :he l$ns are
clear to perc$ssion and a$sc$ltation. E'amination shows a riid a!domen& !owel so$nds are a!sent.
1a!oratory st$dies show:
(emolo!in 1/ 0d1
1e$2ocyte co$nt 1%.2000mm)
<latelet co$nt 1;0.0000mm)
Ser$m
9rea nitroen ,B9N- )/ m0d1
+reatinine 1.* m0d1
:otal !ilir$!in 1.2 m0d1
An '-ray #ilm o# the chest shows a small amo$nt o# #ree air $nder the le#t diaphram. Administration
o# anti!iotics and #l$ids is !e$n. 3hich o# the #ollowin is the most appropriate ne't step in
manaement4
A
- Bari$m swallow
B
- +: scan o# the a!domen
+
- Bntra"eno$s administration o# an (2-receptor !loc2in aent
8
- 9pper endoscopy
E
- 1aparotomy
2*. A ;2-year-old woman with alcoholism comes to the physician a#ter a ser$m cholesterol le"el o#
2*0 m0d1 was #o$nd on a ro$tine screenin. She drin2s a pint o# "od2a daily. She ta2es captopril
#or hypertension and ly!$ride #or type 2 dia!etes mellit$s. She also has intermittent episodes o#
o$t. Fastin ser$m st$dies show:
:otal cholesterol 2;2 m0d1
(81-cholesterol %0 m0d1
:rilycerides )00 m0d1
6l$cose 11% m0d1
:hyroid-stim$latin hormone /.; E90m1
3hich o# the #ollowin is the most appropriate ne't step in manaement4
A
- Alcohol cessation
B
- Better control o# dia!etes
+
- Switch #rom captopril to calci$m-channel !loc2in aent therapy
8
- 6em#i!roDil therapy
E
- :hyroid replacement therapy
)0. A 2)-year-old woman has pain. crampin. and swellin o# the riht cal# ) days a#ter an
$ncomplicated la!or and deli"ery. :he riht #oot is swollen. and there is mar2ed tenderness with
dorsi#le'ion and palpation o# the riht cal#. E'amination shows no other a!normalities. A complete
!lood co$nt and ser$m electrolyte le"els are within normal limits. 3hich o# the #ollowin is the most
li2ely ca$se o# this condition4
A
- (ypercoa$la!le state o# prenancy
B
- (yper$ricemia
+
- <eripheral artery ane$rysm
8
- <latelet em!ol$s
E
- <roloned press$re on the "ena ca"a d$rin deli"ery
)1. A 2-month-old !oy is !ro$ht to the physician #or a well-child e'amination. (e smiles
spontaneo$sly and "ocaliDes witho$t cryin. !$t he does not appear to la$h or sC$eal. (e will not
wor2 #or a toy that is o$t o# his reach. 3hich o# the #ollowin is the most appropriate assessment o#
lan$ae and psychosocial de"elopment4
1an$ae
<sychosocial
de"elopment
de"elopment
A
-
Normal
normal
B
-
Normal
delayed
+
-
8elayed
normal
8
-
8elayed
delayed
)2. A pre"io$sly healthy 2/-year-old woman comes to the physician !eca$se o# a low-rade #e"er
and a nonprod$cti"e co$h #or 7 days. She has !een a!le to contin$e her daily acti"ities. (er
temperat$re is )7.7 + ,**.* F-. A #ew scattered inspiratory crac2les are heard in the thora'. An '-
ray #ilm o# the chest shows patchy in#iltrates in !oth l$ns. 3hich o# the #ollowin is the most
appropriate initial pharmacotherapy4
A
- Amo'icillin
B
- +e#aclor
+
- +ipro#lo'acin
8
- Erythromycin
E
- :rimethoprim-s$l#ametho'aDole
)). A /;-year-old woman comes to the emerency department !eca$se o# shortness o# !reath.
chest pain. diDDiness. and mild n$m!ness and tinlin aro$nd the lips #or 2 ho$rs. She says that she
#eels li2e she is oin to die. She had three similar episodes last wee2 when she was "acationin at
the 6rand +anyon& the #irst episode occ$rred while crossin a narrow !ride on a don2ey. She ta2es
a hypolycemic dr$ #or type 2 dia!etes mellit$s. "erapamil #or hypertension. and s$matriptan as
needed #or miraine. She is mildly diaphoretic and appears pale. (er !lood press$re is 1)00*0 mm
(. p$lse is 1200min. and respirations are 2%0min. Ser$m l$cose le"el is 120 m0d1. An E+6
shows sin$s tachycardia. S$!lin$al nitrolycerin therapy does not relie"e her symptoms and i"es
her a headache. :he most appropriate ne't step in manaement is administration o# which o# the
#ollowin4
A
- (aloperidol
B
- 1oraDepam
+
- ='yen
8
- S$matriptan
E
- Gerapamil
)/. A ?7-year-old man is !ro$ht to the emerency department / ho$rs a#ter the onset o# se"ere
midl$m!ar !ac2 pain. (e is an'io$s. pale. and diaphoretic. (is temperat$re is )7.1 + ,*%.% F-.
!lood press$re is 10;0?; mm (. and p$lse is 1200min. E'amination shows no other a!normalities.
5-ray #ilms o# the l$m!ar spine show deenerati"e disc disease with calci#ications anterior to the
"erte!ral !odies. 3hich o# the #ollowin is t
he most li2ely dianosis4
A
- Aortoiliac occl$sion
B
- (erniated n$cle$s p$lpos$s
+
- 1$m!ar discitis
8
- 1$m!ar strain
E
- <yelonephritis
F
- >$pt$red aortic ane$rysm
6
- Spinal stenosis
);. A 17-year-old !oy is !ro$ht to the emerency department !y his parents !eca$se o# !iDarre
!eha"ior #or ? ho$rs. 1ast niht he was o$t with #riends. and since ret$rnin. he has !een con#$sed
and has RtrashedR his room. (is !lood press$re is 1?;0*; mm (. (e is hyper"iilant. has little
spontaneo$s speech. and is disoriented to place and time. (e appears catatonic !$t a!r$ptly
!ecomes assa$lti"e two times and needs to !e restrained. 3hich o# the #ollowin is the most li2ely
s$!stance ta2en4
A
- +ocaine
B
- Ecstasy
+
- 1S8
8
- MethaC$alone
E
- <+<
:he response options #or the ne't two items are the same. Io$ will !e reC$ired to select one answer
#or each item in the set.
For each patient with !ac2 pain. select the most li2ely dianosis.
A
- (erniated disc
B
- 1$m!ar spinal stenosis
+
- Metastatic cancer
8
- M$scle strain
E
- =steoporotic compression #ract$re
F
- Sacroiliitis
6
- Spinal epid$ral a!scess
(
- Spondylolisthesis
)?. A ;7-year-old woman is !ro$ht to the physician 2 days a#ter the s$dden onset o# se"ere low
!ac2 pain& the pain does not radiate to the lower e'tremities. :he pain !ean when she was li#tin
her randson. She does not ha"e wea2ness or sensory loss in the les and has had no $rinary
incontinence. She has a 10-year history o# rhe$matoid arthritis treated with prednisone. (er
temperat$re is )7 + ,*%.? F-. !lood press$re is 1)00?0 mm (. and p$lse is ?/0min. E'amination
shows de#ormities o# the interphalaneal Joints o# the hands and e'C$isite tenderness to perc$ssion
o"er the l$m!ar spine. Bilateral straiht-le raisin to %0 derees does not increase the pain.
M$scle strenth and sensation are intact in the lower e'tremities. 8eep tendon re#le'es are 2N
!ilaterally. Ba!ins2i7s sin is a!sent !ilaterally.
For each patient with !ac2 pain. select the most li2ely dianosis.
A
- (erniated disc
B
- 1$m!ar spinal stenosis
+
- Metastatic cancer
8
- M$scle strain
E
- =steoporotic compression #ract$re
F
- Sacroiliitis
6
- Spinal epid$ral a!scess
(
- Spondylolisthesis
)7. A pre"io$sly healthy )2-year-old pl$m!er comes to the physician !eca$se o# a )-wee2 history o#
constant. d$ll. low !ac2 pain that does not radiate to the e'tremities. :he pain !ean a#ter he
$nloaded hea"y eC$ipment #rom his "an. Bt increases with acti"ity and is temporarily relie"ed !y
!ed rest and i!$pro#en. E'amination shows tenderness to palpation o"er the l$m!ar paraspinal
reion !ilaterally. :he pain increases with #orward or lateral mo"ements o# the spine. M$scle
strenth and sensation are intact in the lower e'tremities. Bilateral straiht-le raisin to %0
derees does not increase the pain. 8eep tendon re#le'es are 2N !ilaterally. Ba!ins2i7s sin is
a!sent !ilaterally.
)%. A 72-year-old man comes to the physician !eca$se o# a 2-month history o# $rination twice
nihtly and occasional $rinary #reC$ency and $rency. (e has a 1;-year history o# type 2 dia!etes
mellit$s now moderately well controlled with ly!$ride. (is #ather was dianosed with prostate
cancer at the ae o# 70 years. and his sister died o# complications #rom systemic l$p$s
erythematos$s. (is !lood press$re is 1);0%? mm (. +ardiop$lmonary e'amination shows no
a!normalities. A!dominal e'amination shows no s$prap$!ic #$llness or tenderness. :here is mild
enlarement o# the prostate with no palpa!le nod$les. (is post"oid resid$al "ol$me is 10 m1.
Ser$m st$dies show a $rea nitroen ,B9N- le"el o# /; m0d1 and creatinine le"el o# ).% m0d1.
9rine dipstic2 shows )N protein. 3hich o# the #ollowin is most li2ely to ha"e pre"ented proression
o# this patient7s renal disease4
A
- Bntermittent Foley catheteriDation
B
- Bntra"eno$s mannitol therapy
+
- =ral cyclophosphamide and prednisone therapy
8
- =ral enalapril therapy
E
- =ral #inasteride therapy
F
- =ral prednisone therapy only
6
- =ral teraDosin therapy
)*. :wo ho$rs ao. a 2/-year-old man had the s$dden onset o# pain in the riht side o# his chest
that has !ecome increasinly se"ere. (e is now ha"in di##ic$lty !reathin. (is temperat$re. !lood
press$re. and p$lse are normal. An '-ray #ilm o# the chest is shown. 3hich o# the #ollowin is the
most appropriate ne't step in manaement4
A
- Bed rest and sedati"e therapy
B
- Anti!iotic therapy
+
- Anticoa$lant therapy
8
- :$!e thoracostomy
E
- Bmmediate thoracotomy
/0. A 21-year-old collee st$dent comes to the physician !eca$se o# acne that de"eloped / days
ao while she was ta2in her medical collee admission test. She is concerned a!o$t her
appearance and plans to !e in a weddin in ) wee2s. She has had similar episodes that ha"e
resol"ed completely witho$t treatment. E'amination shows ac$te acne o"er the #ace with a
predominance o# comedones and p$st$les. :here is no e"idence o# chronic scarrin. 3hich o# the
#ollowin is the most appropriate initial step in treatment4
A
- 8ietary restriction o# chocolates and simple s$ars
B
- 8ietary restriction o# mil2 prod$cts
+
- :opical acyclo"ir
8
- :opical hydrocortisone cream
E
- :opical retinoic acid
/1. A ?7-year-old woman comes #or a ro$tine health maintenance e'amination. She e'ercises
re$larly. She is not se'$ally acti"e. At her last "isit 1 year ao. her ser$m cholesterol le"el was
1%0 m0d1. and #astin ser$m l$cose le"el was %0 m0d1& a <ap smear and mammoraphy
showed normal #indins. :wo years ao. #le'i!le simoidoscopy showed no a!normalities. :oday.
she weihs ?) 2 ,1/0 l!- and is 1?; cm ,?; in- tall. (er !lood press$re is 1200%0 mm (.
E'amination shows no a!normalities. 3hich o# the #ollowin is the most appropriate screenin test
#or this patient4
A
- <ap smear
B
- Meas$rement o# ser$m cholesterol le"el
+
- Meas$rement o# ser$m l$cose le"el
8
- Mammoraphy
E
- Fle'i!le simoidoscopy
/2. A ?2-year-old man has had the rad$al onset o# #ati$e and shortness o# !reath o"er the past )
years. :here is stri2in J$$lar "eno$s distention with a lare wa"e occ$rrin with S2. :he carotid
$pstro2e is normal. +ardiac e'amination shows a li#tin systolic motion o# the stern$m and no
palpa!le point o# ma'imal imp$lse. A rade )0?. holosystolic. platea$-shaped m$rm$r that is
lo$dest on inspiration is heard at the lower le#t sternal !order. :he li"er is enlared and tender. and
the a!domen is swollen with a #l$id wa"e. :here is mar2ed an2le edema. 3hich o# the #ollowin is
the most li2ely ca$se o# the m$rm$r4
A
- Aortic stenosis
B
- Mitral re$ritation
+
- Mitral stenosis
8
- :ric$spid re$ritation
E
- Gentric$lar septal de#ect
/). A 2/-year-old woman comes to the physician !eca$se o# constant. se"ere pain in her nec2.
sho$lders. and !ac2 #or ) months. She has !een $na!le to enJoy her $s$al acti"ities !eca$se o# the
pain. 9se o# o"er-the-co$nter i!$pro#en and aspirin has not relie"ed her symptoms. She has a
history o# irrita!le !owel syndrome. E'amination shows m$ltiple tender spots o"er the nec2.
sho$lders. and l$m!ar spine. >ane o# motion o# all Joints is #$ll. :here is no e"idence o# syno"itis.
Fl$orescent ser$m antin$clear anti!ody and rhe$matoid #actor assays are neati"e. 3hich o# the
#ollowin is the most li2ely dianosis4
A
- An2ylosin spondylitis
B
- Fi!romyalia
+
- <olymyalia rhe$matica
8
- <olymyositis
E
- Seroneati"e rhe$matoid arthritis
//. A 72-year-old man comes to the physician !eca$se o# a ?-month history o# mild to moderate
shortness o# !reath when clim!in stairs. (e had a myocardial in#arction 2 years ao and has had
an eJection #raction o# );F since then. (is only medication is a M-adreneric !loc2in aent. :he
l$ns are clear to a$sc$ltation. +ardiac e'amination shows an S/ allop. :here is no peripheral
edema. 1a!oratory st$dies are within normal limits. An E+6 shows no ac$te chanes. 3hich o# the
#ollowin is the most appropriate pharmacotherapy4
A
- L-Adreneric !loc2in aent
B
- Aniotensin-con"ertin enDyme ,A+E- inhi!itor
+
- Aniotensin2-receptor !loc2in aent
8
- Nitrates
E
- :hiaDide di$retic
/;. A /2-year-old woman comes to the physician !eca$se o# a )-month history o# a rec$rrent "i"id
dream that se"eral men are assa$ltin her and her children. 9pon awa2enin. she is an'io$s and
distressed !y the #rihtenin imaes. She realiDes that it is J$st a dream !$t is a#raid to o !ac2 to
sleep. She does not 2now why she is ha"in this partic$lar dream !eca$se she has ne"er !een the
"ictim o# an assa$lt. She drin2s two to three c$ps o# co##ee each mornin. She does not $se illicit
dr$s. <hysical e'amination shows no a!normalities. :here is no e"idence o# depressed mood or
hall$cinations. 1a!oratory st$dies are within normal limits. 3hich o# the #ollowin is the most li2ely
dianosis4
A
- Ac$te stress disorder
B
- Nihtmare disorder
+
- <anic disorder
8
- Sleep apnea
E
- Sleep terror disorder
/?. A ;7-year-old man is !ro$ht to the emerency department )0 min$tes a#ter he was #o$nd on
the #loor o# his ho$se& he has le#t hip pain and shortness o# !reath. (e has renal #ail$re !$t has
missed his last two dialysis treatments. (is renal #ail$re was ca$sed !y inad"ertent inestion o#
ethylene lycol. (is renal #$nction did not impro"e. and he is c$rrently on the transplantation list.
Medications incl$de amlodipine and do'aDosin. =n arri"al. his temperat$re is )7.; + ,**.; F-. !lood
press$re is 1;00100 mm (. p$lse is *;0min and re$lar. and respirations are 2/0min. (is
!reathin is rapid and deep. +rac2les are heard in the l$n !ases. E'amination shows a so#t
a!domen. Bowel so$nds are normal. :he le#t lower e'tremity is e'ternally rotated. 1a!oratory
st$dies show:
Ser$m
NaN 1); mEC01
+lA 102 mEC01
KN 7.1 mEC01
(+=)A 12 mEC01
Arterial !lood as analysis on / 10min o# o'yen !y nasal cann$la:
p( 7.22
<+=2 )1 mm (
<=2 ?1 mm (
An E+6 shows pea2ed :-wa"es. Bt will !e at least /; min$tes !e#ore dialysis can !e started. 3hich
o# the #ollowin is the most appropriate ne't step in manaement4
A
- =!ser"ation $ntil dialysis is initiated
B
- Bntra"eno$s calci$m l$conate
+
- Bntra"eno$s l$cose and ins$lin
8
- Bntra"eno$s 0.*F saline
E
- Bntra"eno$s sodi$m !icar!onate
F
- >ectal sodi$m polystyrene s$l#onate ,Kaye'alate-
PPPPPPPPPPPPPPPPPPPPPPPPPPPPPPPPPPPPPPPPPPPPPPPPPPPPPPPPPPPPPPPPPPPPPPPPPPPPPPP
PPPPPPPPPPPPPPPPPPPPPPPPPPPPPPPPPPPPPPPPPPPPPPPPPPP
Section ):--
1. A 2?-year-old woman is !ro$ht to the emerency department !eca$se o# mar2ed con#$sion #or
2 ho$rs& she also has had a #l$-li2e illness #or ) days. ="er the past ? wee2s. she has had increased
#ati$e. wea2ness. and na$sea. She recently started thyroid hormone replacement therapy #or
a$toimm$ne thyroiditis& 1 wee2 ao. her ser$m thyroid-stim$latin hormone le"el was ) E90m1.
(er temperat$re is )% + ,100./ F-. !lood press$re is %00/0 mm (. and p$lse is 1/00min. She
appears con#$sed and letharic. E'amination shows cool. mottled s2in. :here is eneraliDed
hyperpimentation. especially in"ol"in the palmar creases. :he l$ns are clear to a$sc$ltation.
A!dominal e'amination shows di##$se mild tenderness and no re!o$nd. 1a!oratory st$dies show:
(emolo!in 10 0d1
1e$2ocyte co$nt *0000mm)
Semented ne$trophils ;;F
Eosinophils 20F
1ymphocytes 2;F
Ser$m
NaN 12/ mEC01
+lA *2 mEC01
KN ?./ mEC01
(+=)A 1? mEC01
An '-ray #ilm o# the chest and $rinalysis show normal #indins. An E+6 shows sin$s tachycardia with
pea2ed : wa"es. 3hich o# the #ollowin is most li2ely to con#irm the primary ca$se o# this patient7s
condition4
A
- Meas$rement o# p$lmonary artery press$re
B
- Meas$rement o# riht atrial press$re
+
- Meas$rement o# ser$m antithyrolo!$lin anti!ody le"el
8
- Meas$rement o# ser$m lactate dehydroenase acti"ity
E
- Meas$rement o# ser$m thyroid-stim$latin hormone le"el
F
- A+:( stim$lation test
6
- 8e'amethasone s$ppression test
(
- Blood c$lt$res
B
- Echocardioraphy
2. A 27-year-old man comes to the physician !eca$se o# a 1-wee2 history o# shortness o# !reath
with e'ertion. paro'ysmal noct$rnal dyspnea. and swellin o# his #eet. (e has not had chest pain or
palpitations. (e has !een healthy e'cept #or a R!ad coldR 1 month ao that resol"ed spontaneo$sly
a#ter 10 days. (is temperat$re is )7 + ,*%.? F-. !lood press$re is *00?0 mm (. p$lse is 1200min.
and respirations are 2/0min. E'amination shows J$$lar "eno$s distention to % cm. Bilateral !asilar
crac2les are heard. +ardiac e'amination shows a di##$se. laterally displaced point o# ma'imal
imp$lse. :here is a normal S1 and S2 and an S). E'amination shows 2N preti!ial edema !ilaterally.
An E+6 shows no a!normalities. Echocardioraphy is most li2ely to show which o# the #ollowin4
A
- Asymmetric septal hypertrophy
B
- Bic$spid aortic "al"e with stenosis
+
- 8i##$se hypo2inesia and dilation o# the "entricles
8
- 8ys2inesia o# the le#t "entric$lar ape'
E
- Mitral "al"e prolapse
).
A );-year-old man is !ro$ht to the emerency department !eca$se o# intracta!le na$sea and
"omitin o# non!ilio$s #l$id o"er the past /% ho$rs. (e has a history o# d$odenal $lcer disease
treated with (2-receptor !loc2in aents. (is temperat$re is )7 + ,*%.? F-. !lood press$re is *00?0
mm (. p$lse is 1)00min. and respirations are 100min. E'amination shows mild epiastric
tenderness. 3hich o# the #ollowin are the most li2ely ser$m electrolyte #indins4
NaN +lA KN (+=)A
,mEC01- ,mEC01- ,mEC01- ,mEC01-
A
-
11; %0 /.0 2;
B
-
1/0 %0 2.; /0
+
-
1/; 100 ;.0 1;
8
-
1;0 10; 2.; 2;
E
-
1?0 1); ;.0 2;
/. A )-year-old !oy is !ro$ht #or a #ollow-$p e'amination. (e J$st completed a 10-day co$rse o#
amo'icillin that has not resol"ed his riht ear pain. (e appears irrita!le. (is temperat$re is )%.* +
,102 F-. E'amination shows downward and lateral displacement o# the riht a$ricle with tenderness
to palpation o# the posterior a$ric$lar area& his nec2 is s$pple. 3hich o# the #ollowin is the most
appropriate ne't step in dianosis4
A
- Bone scan
B
- +: scan o# the head
+
- :ympanometry
8
- 1$m!ar p$nct$re
E
- :ympanocentesis
;. A ?7-year-old man has had shortness o# !reath on e'ertion #or ) months& he has had an 11.)-2
,2;-l!- weiht loss d$rin this period. (e has smo2ed two pac2s o# ciarettes daily #or 2; years. (e
appears chronically ill. E'amination shows decreased !reath so$nds on the le#t& heart so$nds are
normal. An '-ray #ilm o# the chest shows a lare le#t-sided ple$ral e##$sion. 3hich o# the #ollowin is
the most appropriate ne't step in dianosis4
A
- Bronchoscopy
B
- :horacoscopy
+
- +losed ple$ral !iopsy
8
- =pen ple$ral !iopsy
E
- :horacentesis
?. A proram #or the primary pre"ention o# coronary artery disease is implemented in a comm$nity
in the 9SA. Ass$min that dianostic proced$res and detection remain the same. which o# the
#ollowin meas$res in"ol"in the disease is most e##ecti"e in monitorin the proram4
A
- +ase #atality
B
- (ospitaliDation
+
- Bncidence
8
- Mortality
E
- <re"alence
7. A 77-year-old woman comes to the physician !eca$se o# low !ac2 pain #or ) months. She has
hypertension controlled with a calci$m-channel !loc2in aent and type 2 dia!etes mellit$s
controlled with diet. (er "ital sins are within normal limits. E'amination shows no spinal or
costo"erte!ral anle tenderness& straiht-le raisin prod$ces pain in the low !ac2 at the 12A/
rane. Knee Jer2 and an2le re#le'es are 2N !ilaterally. Ba!ins2i7s sin is a!sent !ilaterally.
9rinalysis shows ;A10 epithelial cells0hp#. 2A; le$2ocytes0hp#. and #ew !acteria. 3hich o# the
#ollowin is the most appropriate pharmacotherapy4
A
- Acetaminophen
B
- 6old
+
- Methotre'ate
8
- <rednisone
E
- <ro!enecid
%. A )2-year-old woman who is (BG positi"e has a +8/N lymphocyte co$nt o# %000mm) ,Normal S
;00-. (er health maintenance reimen sho$ld incl$de imm$niDation aainst which o# the #ollowin
pathoens4
A
- (aemophil$s in#l$enDae type !
B
- (epatitis A
+
- Bn#l$enDa "ir$s
8
- Neisseria meninitidis
E
- Streptococc$s pyoenes ,ro$p A-
*. A )2-year-old woman comes to the physician !eca$se o# !riht red rectal !leedin and se"ere
sta!!in pain with each !owel mo"ement o"er the past 2 wee2s. She has !lood-strea2ed stools.
and there is !lood on the toilet paper. ="er the past 2 months. she has had mild constipation with
no chane in the cali!er o# the stool. E'amination shows a small anal #iss$re at the posterior
midline. >ectal e'amination is pain#$l. !$t no a!normalities are detected e'cept #or a small amo$nt
o# !riht red !lood #rom the #iss$re. 3hich o# the #ollowin is the most appropriate ne't step in
manaement4
A
- Anesthetic ointment and stool so#teners
B
- Anal dilatation $nder anesthesia
+
- 8e!ridement and clos$re o# the #iss$re $nder anesthesia
8
- S$rical #laps
E
- 1ateral internal sphincterotomy
10. A 2%-year-old n$llira"id woman comes #or a ro$tine health maintenance e'amination. She has
had proressi"ely se"ere dysmenorrhea o"er the past ? months adeC$ately controlled !y
nonsteroidal anti-in#lammatory aents. <el"ic e'amination shows a normal "aina and cer"i'. :he
$ter$s is retro"erted and #i'ed. and there is nod$larity o# the c$l-de-sac. A ?-cm le#t adne'al mass
is palpated. :rans"ainal $ltrasonoraphy shows a 7-cm septated adne'al mass. Fo$r wee2s later.
there is no chane in the siDe o# the adne'al mass. 3hich o# the #ollowin is the most appropriate
dianostic test4
A
- Meas$rement o# ser$m +A 12; le"el
B
- Bari$m enema
+
- +: scan o# the pel"is
8
- M>B o# the pel"is
E
- 1aparoscopy
11. A ?7-year-old woman is hospitaliDed !eca$se o# a!dominal pain and persistent copio$s "omitin
#or 2/ ho$rs. :wo wee2s ao. she was hospitaliDed #or treatment o# atrial #i!rillation& a#ter
cardio"ersion to a normal sin$s rhythm. she !ean treatment with war#arin. Iesterday at a #ollow-
$p "isit. her BN> was ?. and her medication was discontin$ed. She ta2es no other medications. (er
temperat$re is )7 + ,*%.? F-. !lood press$re is 10007% mm (. p$lse is 1200min and re$lar. and
respirations are 200min. :he a!domen is distended and moderately tender& there is "ol$ntary
$ardin in the epiastri$m. :here are no masses. oranomealy. or o!"io$s hernias. >ectal
e'amination shows no a!normalities. :est o# the stool #or occ$lt !lood is neati"e. (er hemolo!in
le"el has decreased #rom 1) 0d1 yesterday to 7.% 0d1 today. An E+6 shows a normal sin$s
rhythm. 3hich o# the #ollowin is the most li2ely e'planation #or this patient7s a!dominal symptoms4
A
- Bnternal small-!owel herniation
B
- Bntestinal ischemia #rom a cardiac em!ol$s
+
- Bntram$ral hematoma o# the pro'imal small !owel
8
- Bnt$ss$sception o# the small !owel
E
- Malrotation o# the small !owel
12. A )2-year-old man with alcoholism is !ro$ht to the emerency department !y #riends !eca$se
he has !een $na!le to stand witho$t s$pport and has had R#$nny eye mo"ementsR& they report that
he has !een drin2in appro'imately 1% !eers daily o"er the past month and has !een increasinly
con#$sed o"er the past ; days. (e is awa2e and con#$sed !$t is noncom!ati"e. (is speech is
sl$rred. and his !reath smells o# alcohol. (is temperat$re is )7.2 + ,** F-. !lood press$re is 1%00?0
mm (. p$lse is 1100min. and respirations are 1%0min. <hysical e'amination shows si'th cranial
ner"e palsy. horiDontal diplopia. stra!ism$s. and an asymmetric horiDontal-aDe e"o2ed nystam$s.
Ne$roloic e'amination shows no #ocal wea2ness or n$m!ness. 3hen helped $p and told to wal2.
he has a !road-!ased. $ncertain ait. 3hen as2ed how he arri"ed at the emerency department. he
relates that RB dro"e to this place to "isit some #riends.R :he most li2ely ca$se o# these #indins is a
de#iciency o# which o# the #ollowin4
A
- Folic acid
B
- Manesi$m
+
- Gitamin B1 ,thiamine-
8
- Gitamin B12 ,cyanoco!alamin-
E
- Tinc
1). A )2-year-old woman. ra"ida 2. para 2. comes to the physician !eca$se she has !een
amenorrheic #or / months. E'amination shows a well-estroeniDed "aina and no e"idence o#
"iriliDation or other a!normalities. A ser$m prenancy test is neati"e. She is i"en
medro'yproesterone and has the onset o# !leedin ) days later. 3hich o# the #ollowin is the most
li2ely ca$se o# her condition4
A
- Ano"$lation
B
- Asherman7s syndrome
+
- (ypopit$itarism
8
- Menopa$se
E
- <remat$re o"arian #ail$re
1/.
A pre"io$sly healthy /7-year-old woman comes to the emerency department !eca$se o# a )?-ho$r
history o# na$sea. "omitin. and a!dominal pain that radiates to her !ac2. ="er the past ) years.
she has had intermittent episodes o# crampin a!dominal pain 1 to 2 ho$rs a#ter meals& the pain
lasts #or se"eral ho$rs and resol"es spontaneo$sly. She does not smo2e and drin2s one to two
lasses o# wine each e"enin. :here is a #amily history o# coronary artery disease and hypertension.
(er temperat$re is )7 + ,*%.? F-. !lood press$re is 1000?0 mm (. p$lse is 1200min. and
respirations are 200min. A!dominal e'amination shows moderate epiastric and riht $pper
C$adrant tenderness with no $ardin or re!o$nd& !owel so$nds are decreased. 1a!oratory st$dies
show:
(ematocrit /;F
1e$2ocyte co$nt *0000mm) with a normal di##erential
Ser$m
:otal !ilir$!in 1.; m0d1
Al2aline phosphatase 120 901
Aspartate aminotrans#erase ,AS:. 6=:- 7% 901
Amylase )?; 901
1ipase 122) 901 ,N@1A1?0-
:rilycerides )00 m0d1
3hich o# the #ollowin is the most li2ely dianosis4
A
- Ac$te cholecystitis
B
- Alcoholic hepatitis
+
- Alcoholic pancreatitis
8
- Ascendin cholanitis
E
- 6allstone pancreatitis
F
- (epatitis A
6
- <ancreatic cancer
(
- <eptic $lcer disease
B
- :rilyceride-ind$ced pancreatitis
1;. A healthy )7-year-old primira"id woman at 12 wee2s7 estation comes #or a ro$tine prenatal
"isit. :he prenancy was achie"ed !y in "itro #ertiliDation. She does not $se to!acco. alcohol. or
dr$s. She is a hematoloist and wor2s 10 to 12 ho$rs daily. :wo pre"io$s $ltrasonoraphies ha"e
shown a triplet estation. She weihs ?? 2 ,1/; l!- and is 17% cm ,70 in- tall. (er !lood press$re
is 11?070 mm (. and p$lse is 720min. E'amination shows a $ter$s consistent in siDe with a 1?-
wee2 estation. (er pel"is is normal-siDed. :his patient is at reatest ris2 #or which o# the #ollowin4
A
- A!r$ptio placentae
B
- (epatitis B
+
- (BG in#ection
8
- <reterm la!or
E
- 9terine r$pt$re
1?. A ?7-year-old man is !ro$ht to the emerency department !eca$se o# a )-day history o# #e"er
and headache. Fi"e years ao. he $nderwent placement o# a mechanical aortic "al"e #or treatment
o# seC$elae o# rhe$matic #e"er. (e appears ill. (is temperat$re is /0 + ,10/ F-. !lood press$re is
1100?; mm (. p$lse is 1100min. and respirations are 220min. A rade )0?. systolic eJection
m$rm$r is heard. Ne$roloic e'amination shows mild le#t hemiparesis. Ba!ins2i7s sin is present on
the le#t. :here is no n$chal riidity. :his patient is at reatest ris2 #or which o# the #ollowin
complications4
A
- Brain a!scess
B
- +arotid artery occl$sion
+
- Encephalitis
8
- (ydrocephal$s
E
- Geno$s sin$s throm!osis
17. A ?7-year-old woman comes #or a ro$tine health maintenance e'amination. She drin2s 1 o$nce
o# alcohol daily. (er !lood press$re is 1)%0?2 mm (. p$lse is 7?0min and re$lar. and respirations
are 1/0min. :he l$ns are clear to a$sc$ltation. :he remainder o# the e'amination shows no
a!normalities. 1a!oratory st$dies show:
(emolo!in
12.% 0d1
Ser$m
+a2N
11.* m0d1
+reatinine
0.% m0d1
<hosphor$s
2.% m0d1
:otal protein
?.; 0d1
Al!$min
/.2 0d1
BoniDed calci$m
;.% m0d1 ,N@/.;A;.1-
3hich o# the #ollowin is the most appropriate ne't step in manaement4
A
- Meas$rement o# ser$m al2aline phosphatase acti"ity
B
- Meas$rement o# ser$m parathyroid hormone le"el
+
- Meas$rement o# ser$m "itamin 8 le"el
8
- Ser$m and $rine protein electrophoresis
E
- 5-ray #ilm o# the chest
F
- S2eletal s$r"ey
6
- Bone scan
:he response options #or the ne't two items are the same. Io$ will !e reC$ired to select one answer
#or each item in the set.
For each patient with papilledema. select the most li2ely dianosis.
A
- Bacterial meninitis
B
- +ere!ral in#arction
+
- +ryptococcal meninitis
8
- 6lio!lastoma m$lti#orme
E
- (erpes simple' encephalitis
F
- (ypertensi"e encephalopathy
6
- Bdiopathic intracranial hypertension
(
- Bntracere!ral hemorrhae
B
- St. 1o$is encephalitis
1%. A 2;-year-old woman comes to the emerency department !eca$se o# increasinly se"ere
!i#rontal headaches o"er the past ? months. 8$rin this period. she has had transient episodes o#
!lindness lastin 1 to 2 seconds. She has not had na$sea or "omitin. She has a lon-standin
history o# di##ic$lty losin weiht. She c$rrently weihs 11) 2 ,2;0 l!- and is 1;2 cm ,?0 in- tall.
(er !lood press$re is 1200%0 mm (. Gis$al #ield testin shows enlared !lind spots. :he remainder
o# the ne$roloic e'amination shows normal #indins. A +: scan o# the head with and witho$t
contrast shows no a!normalities. E'amination o# the cere!rospinal #l$id shows:
=penin press$re )00 mm (2=
6l$cose 70 m0d1
<rotein 2; m0d1
3B+ 10mm)
>B+ 00mm)
6ram7s stain and c$lt$res are neati"e.
For each patient with papilledema. select the most li2ely dianosis.
A
- Bacterial meninitis
B
- +ere!ral in#arction
+
- +ryptococcal meninitis
8
- 6lio!lastoma m$lti#orme
E
- (erpes simple' encephalitis
F
- (ypertensi"e encephalopathy
6
- Bdiopathic intracranial hypertension
(
- Bntracere!ral hemorrhae
B
- St. 1o$is encephalitis
1*. A 2;-year-old man with a history o# intra"eno$s dr$ $se comes to the emerency department
!eca$se o# a proressi"e di##$se headache. eneraliDed malaise. and low-rade #e"er #or 2 months.
8$rin this period. he has had a poor appetite res$ltin in a ?.%-2 ,1;-l!- weiht loss. (is
temperat$re is )% + ,100./ F-. E'amination shows UUnec2 sti##ness. Mental stat$s e'amination
shows no a!normalities. +ranial ner"e e'amination shows wea2ness o# the lateral rect$s m$scle on
the riht and !ilateral papilledema. A +: scan o# the head with and witho$t contrast shows
moderate "entric$lar enlarement. E'amination o# cere!rospinal #l$id shows:
=penin press$re 220 mm (2=
6l$cose ); m0d1
<rotein 1;0 m0d1
3B+ 1000mm)
1ymphocytes 100F
>B+ 10mm
20. An asymptomatic )2-year-old woman comes #or a ro$tine health maintenance e'amination. (er
mother and sister ha"e a history o# low !ac2 pain and disc herniation. :he patient is a postal
wor2er. She weihs %? 2 ,1*0 l!- and is 1?; cm ,?; in- tall. (er !lood press$re is 1)007% mm (.
p$lse is 7/0min. and respirations are 1?0min. >ane o# motion o# the spine is normal and witho$t
pain. :here is no scoliosis or e'cessi"e 2yphosis o# the !ac2. 3hich o# the #ollowin is the most
e##ecti"e stratey to decrease this patient7s ris2 #or de"elopin low !ac2 pain4
A
- +hane in Jo!
B
- 1imit physical acti"ity
+
- Stretchin e'ercises
8
- 3eiht-loss proram
E
- Nonsteroidal anti-in#lammatory dr$ therapy
F
- <rophylactic !race
21. :hree days a#ter !ein hospitaliDed #or treatment o# a hip #ract$re s$stained in a #all. a ?2-year-
old woman !ecomes ac$tely short o# !reath and co$hs $p a small amo$nt o# !lood-tined sp$t$m.
She appears an'io$s. (er !lood press$re is 110070 mm (. p$lse is 1100min. and respirations are
2/0min. E'amination shows no other a!normalities. Arterial !lood as analysis on /0F o'yen !y
#ace mas2 shows:
p( 7./0
<+=2 )% mm (
<=2 70 mm (
Gentilation-per#$sion l$n scans show m$ltiple semental areas o# mismatch on the riht. 3hich o#
the #ollowin is the most appropriate ne't step in manaement4
A
- <$lmonary anioraphy
B
- 8opamine therapy
+
- (eparin therapy
8
- 9ro2inase therapy
E
- Bnt$!ation
22. A 2/-year-old man is !ro$ht to the emerency department !y police 1 ho$r a#ter his e'-wi#e
#o$nd him st$m!lin aro$nd in the yard. (is !lood press$re is 100070 mm (. p$lse is *00min. and
respirations are 1?0min. :he p$pils are eC$al and reacti"e to liht& the sclerae are inJected. 8$rin
the e'amination. he la$hs witho$t o!"io$s reason. ma2es reliio$s statements. and as2s i# there is
anythin to eat or drin2. Mental stat$s e'amination shows a !road rane o# a##ect& there is no
e"idence o# tho$ht disorder e'cept #or mild paranoia. 3hich o# the #ollowin is the most
appropriate ne't step in manaement4
A
- =!ser"ation in the emerency department
B
- Bntram$sc$lar administration o# nalo'one
+
- Bntra"eno$s administration o# ;0F de'trose
8
- Bntra"eno$s administration o# loraDepam
E
- =ral administration o# chlorpromaDine
2). A 17-year-old !oy s$stains a head inJ$ry and loses conscio$sness a#ter di"in into a #reshwater
la2e #rom a plat#orm. A#ter !ein s$!mered #or ) min$tes. he is resc$ed. (e is initially cyanotic !$t
then !eins to co$h and !reathe spontaneo$sly a#ter a ;-min$te res$scitation. ="er the ne't )?
ho$rs. which o# the #ollowin complications is most li2ely4
A
- Ac$te respiratory distress syndrome
B
- Bacteremia
+
- Bacterial pne$monia
8
- (ypernatremia
E
- Meta!olic al2alosis
2/. A 1;-year-old !oy is !ro$ht to the emerency department )0 min$tes a#ter !ein in"ol"ed in a
motor "ehicle collision. (e was the $nrestrained passener. =n arri"al. he is disoriented. (is !lood
press$re is %00/0 mm (. p$lse is 1120min. and respirations are 2/0min. :here is no J$$lar
"eno$s distention. E'amination o# the chest shows d$llness to perc$ssion and decreased !reath
so$nds o"er the riht hemithora'. 3hich o# the #ollowin is the most li2ely dianosis4
A
- 8iaphramatic r$pt$re
B
- Flail chest
+
- (emothora'
8
- Massi"e aspiration
E
- <ne$mothora'
2;. An 1%-year-old man comes to the physician #or an initial e'amination !eca$se o# a )-year
history o# #ati$e and miratin Joint pain. (e has !ro$ht a lare #older containin in#ormation
a!o$t pre"io$s medical cons$ltations. la!oratory tests. and '-ray #ilms. (e ta2es no medications. (e
weihs ;0 2 ,110 l!- and is 17) cm ,?% in- tall. <hysical e'amination shows no other
a!normalities. =n mental stat$s e'amination. he is preocc$pied with his symptoms. 3hen as2ed
a!o$t his mood. he states that the #$t$re appears !lea2. and that he is too tired to thin2 a!o$t it.
3hich o# the #ollowin is the most appropriate ne't step in manaement4
A
- As2 a!o$t #$rther symptoms o# o!sessions and comp$lsions
B
- As2 a!o$t se'$al history incl$din se'$al orientation and practices
+
- As2 a!o$t s$icidal #eelins
8
- As2 a!o$t tra"el history o"er the past ? months
E
- =!tain a detailed e'ercise history
F
- Meas$rement o# ser$m 1yme ,Borrelia !$rdor#eri- anti!ody le"el
6
- 9rine to'icoloy screenin
2?. An %0-year-old woman has had !leedin $ms #or ) wee2s. (er diet has consisted o# only tea
and toast #or 2 years. She appears thin and wea2. Gital sins are normal. (er $ms are
hypertrophied and e'$de !lood with press$re. :here are ecchymoses o# the inner thihs and small
hemorrhaes aro$nd the hair #ollicles. :he remainder o# the e'amination shows normal #indins.
S$pplementation with which o# the #ollowin "itamins is most li2ely to ha"e pre"ented this
condition4
A
- A
B
- B12 ,cyanoco!alamin-
+
- +
8
- 8
E
- E
27. A )0-year-old woman. ra"ida 2. para 1. is !ro$ht to the emerency department in la!or. An
episiotomy is per#ormed. Followin deli"ery o# the head. the sho$lders do not #ollow with the $s$al
traction and maternal p$shin. 3hich o# the #ollowin is the most appropriate ne't step in
manaement4
A
- Fle'in the woman7s 2nees toward her sho$lders
B
- More #orce#$l traction and #$ndal press$re
+
- 8eli"erin the posterior arm
8
- >otatin the head 1%0 derees
E
- Symphysiotomy
2%.
A )2-year-old man comes to the physician !eca$se o# a )-day history o# low !ac2 pain. $rinary
hesitancy. and pain with $rination. (e has had se"eral similar episodes o"er the past ) years. (e
has not had any $rethral dischare or recent se'$al contacts. (is temperat$re is )7.2 + ,*%.* F-.
and !lood press$re is 12?07? mm (. :he l$ns are clear to a$sc$ltation& there is no costo"erte!ral
anle tenderness. A!dominal e'amination shows no tenderness or masses. :here is no tenderness
to palpation o# the lower !ac2. Straiht-le raisin to *0 derees is neati"e. :here are no motor or
sensory de#icits in the lower e'tremities. >ectal e'amination shows a di##$se. minimally enlared.
tender prostate with no masses. 1a!oratory st$dies show:
Ser$m prostate-speci#ic antien ? n0m1 ,NV/-
9rine
Speci#ic ra"ity 1.020
Blood neati"e
6l$cose neati"e
Ketones neati"e
1e$2ocyte esterase neati"e
Nitrites neati"e
3hich o# the #ollowin is the most li2ely dianosis4
A
- Benin prostatic hypertrophy
B
- +ystitis
+
- Epididymitis
8
- <rostate cancer
E
- <rostatitis
F
- <yelonephritis
6
- 9rolithiasis
2*. A 1?-year-old !oy with ne$ro#i!romatosis is !ro$ht #or a #ollow-$p e'amination. (is $ncle also
has ne$ro#i!romatosis. (e has a 1-year history o# headaches d$rin which his parents say he
appears pale. Si' months ao. he $nderwent operati"e treatment #or an optic ner"e lioma. (is
!lood press$re is 1?/010; mm (. p$lse is 1020min. and respirations are 1/0min. :he thyroid
lands are not enlared. No m$rm$rs are heard. and radial p$lses are eC$al. A!dominal
e'amination shows no a!normalities. 3hich o# the #ollowin is the most li2ely ca$se o# this patient7s
hih !lood press$re4
A
- +atecholamine-prod$cin t$mor
B
- +arcinoma o# the thyroid land
+
- Essential hypertension
8
- ="erprod$ction o# aldosterone #rom an adrenal adenoma
E
- <osts$!cla"ian coarctation o# the aorta
)0. A 1/-month-old irl is !ro$ht to the physician !eca$se o# a 1/-ho$r history o# irrita!ility and
episodes o# drawin her 2nees toward her chest. 8$rin this period. she has "omited non!ilio$s #l$id
twice and had a !owel mo"ement containin a small amo$nt o# !lood. She had an $pper respiratory
tract in#ection 2 wee2s ao. She is listless e'cept #or intermittent episodes o# discom#ort. (er
temperat$re is )% + ,100./ F-. A!dominal e'amination shows riht-sided tenderness witho$t
$ardin or re!o$nd& !owel so$nds are present. >ectal e'amination shows !riht red !lood and
m$c$s. An '-ray #ilm o# the a!domen shows no a!normalities. 3hich o# the #ollowin is the most
appropriate ne't step in manaement4
A
- 5-ray #ilm o# the $pper astrointestinal tract with contrast
B
- 3ater-sol$!le contrast enema
+
- +orticosteroid enemas
8
- Admission to the hospital #or total parenteral n$trition
E
- Bmmediate laparotomy
)1. An asymptomatic /7-year-old man comes #or a preemployment e'amination. (e has ne"er !een
hospitaliDed. (e is a comp$ter prorammer. and he plays hand!all once wee2ly. (is maternal
randmother had type 2 dia!etes mellit$s. and a paternal $ncle had heart disease. :he patient7s
!lood press$re is 12?0%0 mm (. E'amination shows no a!normalities. (is total ser$m cholesterol
le"el is 22; m0d1. 3hich o# the #ollowin is the most appropriate ne't step in manaement4
A
- Step 2 American (eart Association cardiac diet
B
- Ser$m lipid st$dies while #astin
+
- E'ercise stress test
8
- =ral cholestyramine and niacin therapy
E
- =ral pra"astatin therapy at !edtime
)2. A 2;-year-old woman comes to the physician !eca$se o# a )-month history o# the $ne'plained
$re to eat a #ew ta!lespoons o# cornstarch daily. :he amo$nt o# cornstarch inested has increased
rad$ally d$rin this time. She has leiomyomata $teri& she is otherwise healthy. (er weiht is
$nchaned #rom her last "isit 1 year ao& she weihs ?1 2 ,1); l!- and is 1?% cm ,?? in- tall. (er
!lood press$re is 1200%0 mm (. and p$lse is 1000min. <hysical e'amination shows mild pallor.
Ne$roloic e'amination shows no a!normalities. She is em!arrassed a!o$t her pro!lem. She has no
comp$lsi"e !eha"ior or o!sessi"e tho$hts. An E+6 shows sin$s tachycardia. 3hich o# the #ollowin
is the most appropriate ne't step in manaement4
A
- <sychiatric assessment
B
- +omplete !lood co$nt
+
- :hyroid #$nction st$dies
8
- 9rine to'icoloy screenin
E
- Admission to an eatin disorders clinic
F
- Antipsychotic therapy
6
- Folic acid s$pplementation
(
- Selecti"e serotonin re$pta2e inhi!itor therapy
)). A )0-year-old woman comes to the physician #or a #ollow-$p e'amination / months a#ter
startin treatment with lithi$m car!onate& d$rin this period she has had a ).2-2 ,7-l!- weiht
ain. (er medication was prescri!ed soon a#ter the !irth o# her son to treat racin tho$hts.
increased spendin. and o"ertal2ati"eness& she has had di##ic$lty sleepin #or more than 2 ho$rs
nihtly. (er symptoms decreased a#ter 2 wee2s o# lithi$m car!onate therapy. She contin$es to ta2e
lithi$m car!onate ,)00 m three times daily-. (er temperat$re is )7 + ,*%.? F-. !lood press$re is
1200%0 mm (. and p$lse is 700min. E'amination shows normal #indins. :he most appropriate
ne't step is meas$rement o# which o# the #ollowin4
A
- Fastin ser$m l$cose le"el
B
- 1e$2ocyte co$nt
+
- Ser$m al2aline phosphatase acti"ity
8
- Ser$m creatinine le"el
E
- Ser$m thyroid-stim$latin hormone le"el
)/. =n a ro$tine e'amination. a 2-year-old !oy has a hemolo!in le"el o# 10.; 0d1. hematocrit o#
)0F. and mean corp$sc$lar "ol$me o# 72 Em). (e drin2s #o$r to #i"e %-oD !ottles o# mil2 daily. (e
maintains a re$lar diet !$t does not eat "eeta!les. 3hich o# the #ollowin is the most li2ely
dianosis4
A
- Folic acid de#iciency
B
- Bron de#iciency
+
- Sic2le cell disease
8
- :halassemia
E
- Gitamin B? de#iciency
);. A ;7-year-old man is !ro$ht to the physician !y his wi#e !eca$se his s2in has appeared yellow
#or ) wee2s. E'amination shows Ja$ndice and scleral icter$s. (is total ser$m !ilir$!in le"el is %
m0d1 with a direct component o# ?.2 m0d1. A +: scan o# the a!domen shows a lare lesion in the
head o# the pancreas. 3hen the res$lts are initially disc$ssed. the patient says that he does not
want to hear the report. and his wi#e arees to a!ide !y his wishes. 3hich o# the #ollowin is the
most appropriate co$rse o# action4
A
- 3ithhold the res$lts as the patient wishes
B
- +ontact the patient7s children to disc$ss the res$lts
+
- +ons$lt with the hospital ethics committee
8
- Bnsist on tellin the patient the res$lts
E
- >e#er him to another physician
)?. A pre"io$sly healthy ;2-year-old man comes to the physician !eca$se o# a )-month history o#
increased $rinary "ol$me and increased $rinary #reC$ency at niht. (e has had a ?.%-2 ,1;-l!-
weiht loss d$rin this period despite no chane in appetite. (is #ather has hypertension. and his
mother has hypertension and type 2 dia!etes mellit$s. (e c$rrently weihs *; 2 ,210 l!- and is
17% cm ,70 in- tall. (is !lood press$re is 1?00%; mm ( in !oth arms. E'amination shows no other
a!normalities. (is non#astin ser$m l$cose le"el is 2%0 m0d1. 3hich o# the #ollowin ser$m le"els
is most li2ely to !e increased in this patient4
A
- Bicar!onate
B
- 6l$caon
+
- (81-cholesterol
8
- Bns$lin
E
- Ketones
)7. :hree days a#ter hospitaliDation #or heparin treatment o# deep "eno$s throm!osis o# a le#t
s$per#icial #emoral "ein. a ;2-year-old woman has proloned !leedin #rom a "enip$nct$re site. She
had a p$lmonary em!ol$s 2 years ao. (er temperat$re is )7.; + ,**.; F-. E'amination shows
m$ltiple ecchymoses at the "enip$nct$re sites with ooDin o# #resh !lood. 1a!oratory st$dies show:
(emolo!in
10.; 0d1
<latelet co$nt
2;.0000mm)
<rothrom!in time
1/ sec ,BN>@1.)-
<artial throm!oplastin time
?; sec
<lasma #i!rinoen
)00 m0d1 ,N@200A/00-
Ser$m creatinine
1.1 m0d1
3hich o# the #ollowin is the most li2ely dianosis4
A
- Ad"erse dr$ reaction
B
- 8isseminated intra"asc$lar coa$lation
+
- E'cessi"e !lood loss #rom anticoa$lation
8
- Factor GBBB de#iciency
E
- Factor B5 de#iciency
F
- Bmm$ne throm!ocytopenic p$rp$ra
6
- :hrom!otic throm!ocytopenic p$rp$ra
)%. A )7-year-old woman comes to the physician !eca$se o# an itchy rash o"er her tr$n2 #or 2
wee2s. She has not had #e"er. chills. shortness o# !reath. chest pain. or astrointestinal symptoms.
She has a history o# rec$rrent $rinary tract in#ections and has !een ta2in trimethoprim-
s$l#ametho'aDole prophyla'is #or the past year. She is in mild distress. (er temperat$re is )7.; +
,**.; F-. !lood press$re is *?0?2 mm (. p$lse is 7%0min. and respirations are 1/0min.
E'amination shows a mac$lopap$lar erythemato$s rash o"er the tr$n2. 1a!oratory st$dies show:
1e$2ocyte co$nt 10.;000mm)
Semented ne$trophils 72F
Bands 1F
Eosinophils 1;F
1ymphocytes /F
Monocytes %F
Ser$m
9rea nitroen ,B9N- 12 m0d1
+reatinine 0.* m0d1
9rine
3B+ 20hp#
>B+ 20hp#
3hich o# the #ollowin is the most li2ely ca$se o# these #indins4
A
- EcDema
B
- Medication ad"erse e##ect
+
- Staphylococcal s2in in#ection
8
- Streptococcal s2in in#ection
E
- 9rinary tract in#ection
)*. A 1*-year-old primira"id woman at )/ wee2s7 estation comes to the physician #or a ro$tine
prenatal "isit. (er prenancy has !een $ncomplicated. She has no history o# serio$s illness. She
ta2es no medications and has no 2nown alleries. E'amination shows a $ter$s consistent in siDe
with a )/-wee2 estation. A ro$tine clean-catch $rine c$lt$re rows reater than 100.000
colonies0m1 o# Escherichia coli. 3hich o# the #ollowin is the most appropriate pharmacotherapy4
A
- Ampicillin
B
- +ipro#lo'acin
+
- +lindamycin
8
- 8o'ycycline
E
- :rimethoprim-s$l#ametho'aDole
/0. A 1)-year-old irl is !ro$ht #or a well-child e'amination. Menses ha"e occ$rred e"ery other
month since menarche 10 months ao. (er last menstr$al period was 1 wee2 ao. She is not
se'$ally acti"e. Se'$al de"elopment is :anner stae ). E'amination shows no a!normalities. 3hich
o# the #ollowin is the most appropriate ne't step in manaement4
A
- 8isc$ssion o# prenancy pre"ention
B
- Meas$rement o# ser$m l$teiniDin and #ollicle-stim$latin hormone le"els
+
- 9rine M-h+6 test
8
- <el"ic e'amination
E
- Estroen therapy
F
- 8ianostic laparoscopy
/1. A 72-year-old man comes to the emerency department a#ter a ;-min$te episode o# !lindness
in the riht eye. ="er the past month. he has had headache and pain in the Jaw with chewin. (is
temperat$re is )%.1 + ,100.? F-. E'amination shows tender. nod$lar temporal arteries with
decreased p$lses. (is erythrocyte sedimentation rate is *2 mm0h. 3hich o# the #ollowin is the most
appropriate immediate step in manaement4
A
- +arotid $ltrasonoraphy
B
- Aspirin therapy
+
- +e#ota'ime therapy
8
- +orticosteroid therapy
E
- :emporal artery !iopsy
/2. A /2-year-old man comes #or a ro$tine health maintenance e'amination. :here is no #amily
history o# coronary artery disease. and he does not smo2e. (is weiht is appropriate #or his heiht.
(is !lood press$re is 1200%0 mm (. Ser$m lipid st$dies show a total cholesterol le"el o# 1*0
m0d1. (81-cholesterol le"el o# /0 m0d1. and trilyceride le"el o# 1;0 m0d1. 3hich o# the
#ollowin is the most appropriate ne't step in manaement4
A
- >ecommend the Step 2 National +holesterol Ed$cation <roram diet
B
- Meas$re ser$m 181-cholesterol le"el now
+
- Meas$re total ser$m cholesterol le"el in ; years
8
- <rescri!e prophylactic aspirin
E
- Bein treatment with lo"astatin
/). A 2%-month-old !oy has a history o# cyanosis since !irth with episodes o# syncope. E'amination
shows cyanosis and cl$!!in. :he l$ns are clear to a$sc$ltation. :here is a riht "entric$lar hea"e.
a systolic clic2. a sinle S2. and a rade )0? systolic m$rm$r. 3hich o# the #ollowin is the most
li2ely dianosis4
A
- Bic$spid aortic "al"e
B
- +oarctation o# the aorta
+
- Mitral stenosis
8
- <atent d$ct$s arterios$s
E
- :etraloy o# Fallot
//. A healthy 1%-year-old woman comes #or a ro$tine health maintenance e'amination. Menses
occ$r at re$lar 2%-day inter"als. (er last menstr$al period was 2 wee2s ao. She is se'$ally acti"e
with one partner. and she and her partner $se condoms inconsistently #or contraception. (er
maternal randmother was dianosed with !reast cancer at the ae o# ?; years. her paternal
rand#ather was dianosed with colon cancer at the ae o# 72 years. and her maternal rand#ather
died o# a myocardial in#arction at the ae o# ?? years. E'amination shows no a!normalities. 3hich
o# the #ollowin is the most appropriate screenin test #or this patient4
A
- +omplete !lood co$nt
B
- Fastin ser$m lipid st$dies
+
- :est o# the stool #or occ$lt !lood
8
- :estin #or Neisseria onorrhoeae and +hlamydia trachomatis
E
- 9rinalysis
:he response options #or the ne't two items are the same. Io$ will !e reC$ired to select one answer
#or each item in the set.
For each child with #e"er and co$h. select the most li2ely dianosis.
A
- L1-Antitrypsin de#iciency
B
- +ystic #i!rosis
+
- <ne$mothora'
8
- <$lmonary al"eolar proteinosis
E
- <$lmonary asperillosis
F
- <$lmonary hemorrhae
6
- <$lmonary t$!erc$losis
/;. A 20-month-old irl is !ro$ht to the physician !eca$se o# #e"er and co$h #or 2 days. She has
had se"eral similar episodes since the ae o# / months. :hree months ao. she and her #amily
"isited her randmother in Finland #or 2 wee2s. She is at the 2;th percentile #or lenth and ;th
percentile #or weiht. She appears thin and pale. (er temperat$re is )% + ,100./ F-. p$lse is
1;00min. and respirations are /00min. E'amination shows mild cl$!!in. 3heeDin and !ilateral
crac2les are heard at the l$n !ases. An '-ray #ilm o# the chest shows strea2y densities !ilaterally
with mild hyperin#lation.
For each child with #e"er and co$h. select the most li2ely dianosis.
A
- L1-Antitrypsin de#iciency
B
- +ystic #i!rosis
+
- <ne$mothora'
8
- <$lmonary al"eolar proteinosis
E
- <$lmonary asperillosis
F
- <$lmonary hemorrhae
6
- <$lmonary t$!erc$losis
/?. A pre"io$sly healthy 1?-year-old !oy is !ro$ht to the physician !eca$se o# #e"er and co$h
with riht-sided chest pain #or 2 wee2s. Si' months ao. he "isited his randparents in Al!ania #or 2
wee2s. (e weihs ;/ 2 ,120 l!- and is 17) cm ,?% in- tall. (e appears thin and pale. (is
temperat$re is )%.2 + ,100.% F-. p$lse is 7?0min. and respirations are )?0min. E'amination shows
shallow respirations with decreased !reath so$nds at the riht l$n !ase. An '-ray #ilm o# the chest
shows a riht ple$ral e##$sion and hilar adenopathy.
PPPPPPPPPPPPPPPPPPPPPPPPPPPPPPPPPPPPPPPPPPPPPPPPPPPPPPPPPPPPPPPPPPPPPPPPPPPPPPP
PPPPPPPPPPPPPPPPPPPPPPPPPPPPPPPPPPPPPPPPPPPPPPPPPPP
Section /:--
1. A pre"io$sly healthy ;2-year-old man comes to the emerency department !eca$se o# hicc$ps
#or 1 wee2. (e has smo2ed two pac2s o# ciarettes daily #or )0 years. (e does not drin2 alcohol. (e
is alert and oriented. (is temperat$re is )7 + ,*%.? F-. !lood press$re is 1;00*; mm (. p$lse is
700min. and respirations are 120min. <hysical and ne$roloic e'aminations show no a!normalities.
(is ser$m sodi$m le"el is 120 mEC01. An '-ray #ilm o# the chest shows a riht hilar mass. 3hich o#
the #ollowin is the most appropriate ne't step in treatment4
A
- Bisphosphonate therapy
B
- +alcitonin therapy
+
- +alci$m therapy
8
- 8e'amethasone therapy
E
- ;F 8e'trose in 0.22;F saline therapy
F
- ;F 8e'trose in 0./;F saline therapy
6
- ;F 8e'trose in water therapy
(
- Fl$id restriction
B
- (ydrocortisone therapy
H
- 1actated >iner7s sol$tion
K
- Mannitol therapy
1
- <otassi$m therapy
M
- 0.*F Saline therapy
N
- )F Saline therapy
=
- Sodi$m !icar!onate therapy
2. A ?7-year-old man comes to the physician !eca$se o# a ?-month history o# do$!le "ision. sl$rred
speech. and di##ic$lty swallowin liC$ids. Bnitially. his symptoms were intermittent !$t now occ$r
daily and are worse at the end o# the day. ="er the past wee2. he has had shortness o# !reath with
e'ertion. (e has pernicio$s anemia treated with monthly "itamin B12 ,cyanoco!alamin- inJections
and a$toimm$ne thyroid disease c$rrently treated with thyroid replacement therapy. E'amination
shows !ilateral ptosis and disconJ$ate aDe. :here is !ilateral #acial wea2ness and hypernasal
speech. :he ton$e is wea2. and the a re#le' is red$ced. M$scle strenth is /0; in the pro'imal
m$scles o# the $pper and lower e'tremities. 8eep tendon re#le'es are 2N di##$sely. Ba!ins2i7s sin
is a!sent. Sensory e'amination shows no a!normalities. A +: scan o# the chest is shown. 3hich o#
the #ollowin is the most li2ely dianosis o# this patient7s intrathoracic lesion4
A
- Aspiration pne$monia
B
- Bronchoenic carcinoma
+
- 1$n metastasis
8
- Sarcoidosis
E
- :hymoma
F
- :hyroid carcinoma
). A )2-year-old man recei"in intensi"e chemotherapy #or (od2in7s disease has a temperat$re o#
)* + ,102.2 F-. (is respirations are /00min. 3idespread crac2les are heard in all l$n #ields. An '-
ray #ilm o# the chest shows a di##$se al"eolar and interstitial pattern. 3hich o# the #ollowin is the
most li2ely ca$sal oranism4
A
- Asperill$s species
B
- +andida al!icans
+
- +occidioides immitis
8
- <ne$mocystis carinii
E
- Streptococc$s pne$moniae
/. A 20-year-old A#rican American man with sic2le cell disease comes to the physician !eca$se o# a
1-wee2 history o# shortness o# !reath on e'ertion. #ati$e. and eneraliDed wea2ness. (e has had
no #e"er. chills. niht sweats. or co$h prod$cti"e o# sp$t$m. (is only medication is o'ycodone #or
Joint pain. (e weihs ?% 2 ,1;0 l!- and is 1?% cm ,?? in- tall. (is temperat$re is )?.7 + ,*% F-.
!lood press$re is 120070 mm (. p$lse is 7?0min. and respirations are 1%0min. E'amination shows
no a!normalities. 1a!oratory st$dies show:
(ematocrit 20F
Mean corp$sc$lar "ol$me 110 Em)
1e$2ocyte co$nt 2)000mm)
>etic$locyte co$nt 1.%F
3hich o# the #ollowin is the most li2ely mechanism #or these #indins4
A
- Ad"erse dr$ reaction
B
- Atrophy o# astric m$cosa
+
- Bacterial o"errowth in the small intestine
8
- Bncreased demand #or #olic acid
E
- Bncreased demand #or "itamin B12 ,cyanoco!alamin-
F
- Mala!sorption
6
- Gitamin B1 ,thiamine- de#iciency
;. A //-year-old woman comes to the emerency department 2 days a#ter !ein dischared #rom
the hospital #or a!dominal pain& an e'ploratory laparoscopy showed no a!normalities. ="er the past
; years. she had !een admitted to the hospital n$mero$s times #or the e"al$ation o# a "ariety o#
symptoms& all wor2-$ps had !een neati"e. (er temperat$re is )*.2 + ,102.? F-. E'amination
shows a reddened. ind$rated laparoscopic wo$nd. She is admitted to the hospital and i"en
intra"eno$s anti!iotics. =n the second hospital day. a n$rse witnesses the patient r$!!in sali"a
into her laparoscopy site. 3hich o# the #ollowin is the most li2ely dianosis4
A
- AdJ$stment disorder with dist$r!ance o# cond$ct
B
- +ond$ct disorder
+
- +on"ersion disorder
8
- Factitio$s disorder
E
- (ypochondriasis
F
- MaJor depressi"e disorder
6
- =ppositional de#iant disorder
(
- SomatiDation disorder
?. A )7-year-old woman is !ro$ht to the emerency department !eca$se she has !een $na!le to
see o$t o# her riht eye since awa2enin 2 ho$rs ao. She states that any mo"ement o# the eye is
pain#$l. E'amination shows "is$al ac$ity o# 200200 in the riht eye and 20020 in the le#t eye. :he
optic #$ndi are normal. :he le#t p$pil reacts normally to liht. :he riht p$pil is poorly reacti"e to
direct liht. :he remainder o# the eye e'amination shows no a!normalities. 3hich o# the #ollowin is
the most li2ely site o# the lesion4
A
- 1e#t optic ner"e
B
- 1e#t optic radiation
+
- 1e#t optic tract
8
- 1e#t "is$al corte'
E
- =ptic chiasm
F
- >etina
6
- >iht optic ner"e
(
- >iht optic radiation
B
- >iht optic tract
H
- >iht "is$al corte'
:he response options #or the ne't two items are the same. Io$ will !e reC$ired to select one answer
#or each item in the set.
For each patient with peripheral ner"e dys#$nction. select the most li2ely site o# ner"e inJ$ry.
A
- A'illary ner"e
B
- +er"ical ner"e root at the cer"ical #oramen
+
- 1on thoracic ner"e
8
- Median ner"e a!o"e the el!ow
E
- Median ner"e at the el!ow
F
- Median ner"e at the wrist
6
- M$sc$loc$taneo$s ner"e a!o"e the el!ow
(
- M$sc$loc$taneo$s ner"e at the el!ow
B
- M$sc$loc$taneo$s ner"e at the wrist
H
- >adial ner"e a!o"e the el!ow
K
- >adial ner"e at the el!ow
1
- >adial ner"e at the wrist
M
- S$prascap$lar ner"e
N
- :horacodorsal ner"e
=
- 9lnar ner"e a!o"e the el!ow
<
- 9lnar ner"e at the el!ow
O
- 9lnar ner"e at the wrist
7.
A pre"io$sly healthy /2-year-old carpenter comes to the physician !eca$se o# a ?-month history o#
pain and n$m!ness in his dominant hand that awa2ens him at niht. (e descri!es n$m!ness in his
lon and inde' #iners a#ter dri"in #or e'tended periods o# time. E'amination shows minimal
atrophy o# the thenar m$scles. :here is normal sensation to liht to$ch on the little #iner and the
palm o# the riht hand. Sensation to liht to$ch is decreased at the tip o# the th$m!. inde' #iner.
and lon #iner. 5-ray #ilms o# the riht el!ow and wrist show no a!normalities.
For each patient with peripheral ner"e dys#$nction. select the most li2ely site o# ner"e inJ$ry.
A
- A'illary ner"e
B
- +er"ical ner"e root at the cer"ical #oramen
+
- 1on thoracic ner"e
8
- Median ner"e a!o"e the el!ow
E
- Median ner"e at the el!ow
F
- Median ner"e at the wrist
6
- M$sc$loc$taneo$s ner"e a!o"e the el!ow
(
- M$sc$loc$taneo$s ner"e at the el!ow
B
- M$sc$loc$taneo$s ner"e at the wrist
H
- >adial ner"e a!o"e the el!ow
K
- >adial ner"e at the el!ow
1
- >adial ner"e at the wrist
M
- S$prascap$lar ner"e
N
- :horacodorsal ner"e
=
- 9lnar ner"e a!o"e the el!ow
<
- 9lnar ner"e at the el!ow
O
- 9lnar ner"e at the wrist
%. A pre"io$sly healthy )7-year-old man comes to the physician !eca$se o# a 2-month history o#
pain in the #orearm and little #iner o# his dominant hand& he has !een wor2in as a receptionist #or
? months. (e descri!es n$m!ness in his little #iner and wea2ness o# his rip. :here is decreased
sensation to liht to$ch at the tip o# the little #iner. <aresthesias are elicited with compression o#
the c$!ital t$nnel.
*. A )-month-old !oy is !ro$ht #or a well-child e'amination. (e has poor head control.
E'amination shows eneraliDed hypotonia. :he point o# ma'imal imp$lse is at the le#t anterior
a'illary line. :he li"er ede is palpated / cm !elow the riht costal marin. :he spleen is not
palpa!le. 3hich o# the #ollowin is the most li2ely dianosis4
A
- +onenital m$sc$lar dystrophy
B
- 6lycoen storae disease. type BB ,<ompe7s disease-
+
- 6M1 anliosidosis
8
- Bn#ant !ot$lism
E
- Gentric$lar septal de#ect
10. ="er the past ) months. a )0-year-old woman has had intermittent episodes o# headache.
palpitations. sweatin. and irrita!ility. (er !lood press$re is 1;00100 mm ( while s$pine and
1/*0100 mm ( while standin. and p$lse is *00min while s$pine and 1100min while standin.
E'amination shows no a!normalities e'cept #or pallor. 3hich o# the #ollowin is the most li2ely
location o# this patient7s lesion4
A
- Adrenal land
B
- Brain
+
- (eart
8
- Kidney
E
- :hyroid land
11. A 1*-year-old man is !ro$ht to the emerency department /; min$tes a#ter s$stainin a
sinle. lare sta! wo$nd to the riht $pper C$adrant o# the a!domen. (e is o!t$nded. (is !lood
press$re is ?00palpa!le mm (. and p$lse is 1/%0min. Breath so$nds are eC$al !ilaterally.
E'amination shows a /-cm laceration in the riht $pper C$adrant in the midcla"ic$lar line. :he
a!domen is distended. 3hich o# the #ollowin is the most appropriate ne't step in manaement4
A
- 5-ray #ilms o# the a!domen and pel"is
B
- Anioraphy
+
- +ontrast st$dy o# the wo$nd tract
8
- +: scan o# the a!domen
E
- +: scan o# the chest
F
- 1aparoscopy
6
- <eritoneal la"ae
(
- 1aparotomy
B
- 1ocal wo$nd e'ploration
12. A ;2-year-old woman comes to the physician !eca$se o# decreased li!ido& this symptom !ean
% months ao. a#ter she $nderwent a total a!dominal hysterectomy and !ilateral salpino-
oophorectomy #or leiomyomata $teri and menorrhaia. She has !een ta2in hormone replacement
therapy with conJ$ated estroen since the operation. E'amination shows a moist. r$ated "aina.
3hich o# the #ollowin is the most li2ely ca$se o# these #indins4
A
- 8ecreased androens
B
- 8ecreased estroen
+
- 8ecreased #ollicle-stim$latin hormone ,FS(-
8
- 8ecreased l$teiniDin hormone ,1(-
E
- 8ecreased proesterone
F
- 8ecreased prolactin
6
- Bncreased androens
(
- Bncreased estroen
B
- Bncreased FS(
H
- Bncreased 1(
K
- Bncreased proesterone
1
- Bncreased prolactin
1).
A 7-day-old new!orn is !ro$ht #or a well-child e'amination. (e was !orn at home. (is mother has
ta2en acetaminophen #or perineal discom#ort. :he new!orn is !reast-#eedin well. (e is at the 7;th
percentile #or lenth and weiht. E'amination shows no a!normalities. A#ter "accination with
hepatitis B. he has proloned !leedin at the inJection site. :here is no #amily history o# e'cessi"e
!leedin. 1a!oratory st$dies show:
(emolo!in 1? 0d1
1e$2ocyte co$nt ?%000mm)
Semented ne$trophils /%F
Bands 2F
1ymphocytes ;0F
<latelet co$nt 2%0.0000mm)
<rothrom!in time 20 sec ,BN>@1.?-
<artial throm!oplastin time ?0 sec
3hich o# the #ollowin is the most li2ely mechanism #or these #indins4
A
- A$toimm$niDation
B
- Bacterial to'ic e##ect
+
- Factor GBBB de#iciency
8
- Factor B5 de#iciency
E
- Bmm$nolo!$lin de#iciency
F
- Bron de#iciency
6
- <harmacoloic e##ect
(
- Giral to'ic e##ect
B
- Gitamin de#iciency
H
- Tinc de#iciency
1/.
A ?7-year-old man with lon-standin sins and symptoms o# conesti"e heart #ail$re is admitted to
the hospital !eca$se o# proressi"e shortness o# !reath. E'amination shows no other a!normalities.
An '-ray #ilm o# the chest shows cardiomealy. cephaliDation o# !lood "essels. and a riht-sided
ple$ral e##$sion. 3hich o# the #ollowin sets o# ple$ral #l$id #indins is most li2ely in this patient4
1e$2ocyte Semented
<rotein 6l$cose co$nt ne$trophils Monocytes
,0d1- ,m0d1- ,0mm)- ,F- ,F-
A
-
2.;
10
10.000
;0
;0
B
-
2.;
*0
2000
?0
/0
+
-
).%
/0
)0.000
%0
20
8
-
/.;
?0
10.000
20
%0
E
-
/.;
*0
2000
;0
;0
1;. For % wee2s. a ;2-year-old man with a ;-year history o# type 2 dia!etes mellit$s has had deep
!$rnin pain in the !all o# his riht #oot and !i toe when the #oot is raised a!o"e chest le"el. (e
also has crampin in his riht cal# when he wal2s more than ;0 #eet. (e has smo2ed two pac2s o#
ciarettes daily #or )0 years. Femoral p$lses are palpa!le& pedal p$lses are a!sent. 3hich o# the
#ollowin is the most li2ely dianosis4
A
- Aortoiliac stenosis
B
- Femoral popliteal stenosis
+
- Monone$ropathy
8
- Gasc$litis
E
- Geno$s stasis
1?. A 1-year st$dy o# a new dr$ to treat hypertension is cond$cted. =ne h$ndred patients with
hypertension are enrolled& ;0 patients are i"en the new dr$ and another ;0 patients are i"en
hydrochlorothiaDide. All patients completed the trial. =ne noted $ne'pected e##ect is increased
rowth o# scalp hair which occ$rred in those ta2in the new dr$. a nonstatistically sini#icant
di##erence ,pW0.10-. :his e##ect has also !een reported in st$dies o# other similar dr$s in the new
therape$tic class. :he in"estiators o# the st$dy concl$ded that the new dr$ did not ca$se hair
rowth. 3hich o# the #ollowin #eat$res o# this st$dy is most li2ely to a##ect the "alidity o# this
concl$sion4
A
- 8i##erential #ollow-$p
B
- 1ead time !ias
+
- 1enth o# the st$dy
8
- Sample siDe
E
- Sel#-selection
17. A )%-year-old woman. ra"ida 2. para 1. at )% wee2s7 estation has had no #etal mo"ement #or
)? ho$rs. (er prenatal co$rse. prenatal tests. and #etal rowth ha"e !een normal. Fetal heart tones
are heard !y 8oppler. 3hich o# the #ollowin is the most appropriate ne't step in manaement4
A
- >o$tine prenatal "isit in 1 wee2
B
- Maternal hydration
+
- Nonstress test
8
- Bmmediate ind$ction o# la!or
E
- Amniocentesis
1%. A healthy 7-year-old !oy is !ro$ht to the physician 1 wee2 a#ter he was e'posed #or se"eral
ho$rs to a child with chic2enpo'. :he patient and his healthy sister ha"e not had chic2enpo'. :hey
ha"e not recei"ed "aricella "accine. 3hich o# the #ollowin is the most appropriate manaement #or
the patient and his sister at this time4
A
- Administer acyclo"ir as prophyla'is
B
- Administer imm$ne lo!$lin. intra"eno$sly
+
- Administer aspirin therapy i# "esicles appear
8
- Ad"ise the parents to 2eep the si!lins home #rom school to pre"ent e'posin their classmates
E
- Bn#orm the parents that a "esic$lar rash may appear at any time o"er the ne't 2 wee2s
1*. A ?-year-old !oy is !ro$ht to the physician !y his mother !eca$se o# proressi"e "is$al loss
o"er the past year. ="er the past 2 years. he has had deterioration o# his hearin. speech. writin.
and intellect$al per#ormance. (is maternal $ncle had similar symptoms. Gis$al ac$ity is 200200
!ilaterally. F$nd$scopic e'amination shows optic atrophy. (is hearin is mar2edly impaired. :here is
wea2ness and spasticity o# all e'tremities. 8eep tendon re#le'es are e'tremely hyperacti"e.
Ba!ins2i7s sin is present !ilaterally. =n mental stat$s e'amination. he is not oriented to place.
year. month. or the names o# his si!lins. An M>B o# the !rain shows mar2ed symmetric white
matter disease in"ol"in all lo!es. 8ianostic st$dies are most li2ely to show which o# the #ollowin4
A
- A!normally decreased ser$m cholesterol le"el
B
- Acanthocytes on !lood smear
+
- An e'cess o# "ery lon chain #atty acids
8
- Normal ner"e cond$ction st$dies
E
- Gitamin E de#iciency
20. A 22-year-old primira"id woman at 1? wee2s7 estation is !ro$ht to the emerency
department !eca$se o# proressi"e shortness o# !reath o"er the past /% ho$rs. (er temperat$re is
)7 + ,*%.? F-. !lood press$re is 120070 mm (. p$lse is 1000min. and respirations are 2/0min.
Scattered wheeDes are heard. <el"ic e'amination shows a $ter$s that e'tends to the $m!ilic$s.
Fetal heart tones are a!sent. (er hematocrit is )2F. le$2ocyte co$nt is 11.0000mm). and ser$m M-
h+6 le"el is )00.000 mB90m1. <$lse o'imetry on room air shows an o'yen sat$ration o# *2F. An
'-ray #ilm o# the chest shows m$ltiple ro$nd densities thro$ho$t all l$n #ields. 3hich o# the
#ollowin is the most li2ely dianosis4
A
- Bacterial pne$monia
B
- +horiocarcinoma
+
- <$lmonary em!olism
8
- :$!erc$losis
E
- Giral pne$monia
21. A /2-year-old man comes to the physician !eca$se o# proressi"e swellin o# the les o"er the
past 2 months. (e has a history o# stae BBA (od2in7s disease treated 1 year ao with radiation
therapy to the nec2 and chest. (is temperat$re is )7 + ,*%.? F-. !lood press$re is 1020%0 mm (.
p$lse is 1100min. and respirations are 220min. E'amination shows J$$lar "eno$s distention that
increases with inspiration. :he l$ns are clear to a$sc$ltation. +ardiac e'amination shows a
nondisplaced point o# ma'imal imp$lse& heart so$nds are distant. An early diastolic so$nd is heard
at the ape'. A!dominal e'amination shows mild distention with shi#tin d$llness. :he li"er is
p$lsatile. and its ede is palpated / cm !elow the riht costal marin. :here is 2N peripheral edema
e'tendin $p to the 2nees. 3hich o# the #ollowin is the most li2ely mechanism o# this patient7s
increased central "eno$s press$re4
A
- +onstricti"e pericarditis
B
- +or p$lmonale
+
- 1e#t-sided conesti"e heart #ail$re
8
- Mitral stenosis
E
- :ric$spid stenosis
22. A ;-year-old !oy is !ro$ht to the emerency department )0 min$tes a#ter he #ainted at home
a#ter standin $p #rom a sittin position. (is symptoms !ean ) days ao with diarrhea and
"omitin. (e has had no $rine o$tp$t #or 1% ho$rs. (e is alert !$t C$iet. (is temperat$re is )7.; +
,**.; F-. !lood press$re is 7;0/; mm (. p$lse is 1200min. and respirations are 2%0min.
E'amination shows dry lips and tentin o# the s2in. :here is no a!dominal tenderness. Bowel so$nds
are hyperacti"e. :he remainder o# the e'amination shows no a!normalities. (is capillary re#ill time
is ; seconds. Bntra"eno$s !ol$s doses o# 0.*F saline are administered. Bladder catheteriDation
yields ; m1 o# $rine. 9rinalysis is most li2ely to show which o# the #ollowin4
A
- Blood
B
- Erythrocyte casts
+
- (yaline casts
8
- 1e$2ocyte casts
E
- ='alate crystals
2).
A 2-year-old !oy is !ro$ht to the physician !eca$se o# #e"er and co$h #or 2 days. (e had
Streptococc$s pne$moniae meninitis at the ae o# 1 year. S. pne$moniae !acteremia at the ae o#
1% months. and pne$monia at the ae o# 22 months. :wo maternal $ncles died !e#ore the ae o# 2
years #rom Rin#ection.R (is temperat$re is )*.% + ,10).? F-. p$lse is 1;00min. and respirations are
?00min. E'amination shows s$!costal retractions on inspiration. 1a!oratory st$dies show:
(emolo!in 10 0d1
1e$2ocyte co$nt )?.0000mm)
Semented ne$trophils 70F
Bands 20F
1ymphocytes %F
Monocytes 2F
<latelet co$nt 2/0.0000mm)
Ser$m
BA V; m0d1
B6 )0 m0d1
BM V; m0d1
An '-ray #ilm o# the chest shows an in#iltrate in the le#t $pper lo!e. 3hich o# the #ollowin is the
most appropriate ne't step in manaement4
A
- >ee'amination in 12 wee2s
B
- <rednisone therapy only
+
- <rednisone. "incristine. do'or$!icin. asparainase. and methotre'ate therapy
8
- Tido"$dine ,AT:-. lami"$dine ,):+-. and ritona"ir therapy
E
- Bntra"eno$s imm$nolo!$lin in#$sion
F
- Bone marrow transplantation
6
- :hym$s transplantation
2/.
A pre"io$sly healthy 1)-year-old irl is !ro$ht to the physician !eca$se o# a 2-month history o#
intermittent a!dominal pain and loose stools. She has had a 2.)-2 ,;-l!- weiht loss d$rin this
period d$e to a decreased appetite. !$t she drin2s $p to 1 liter o# #r$it J$ice daily. At her last "isit *
months ao. she was at the
;0th percentile #or heiht and the ;0th percentile #or weiht. She is now at the ;0th percentile #or
heiht and the 10th percentile #or weiht. (er temperat$re is )7.% + ,100 F-. p$lse is %00min. and
respirations are 1%0min. E'amination shows no other a!normalities. :est o# the stool #or occ$lt
!lood is positi"e. 1a!oratory st$dies show:
(emolo!in %.; 0d1
1e$2ocyte co$nt *1000mm)
Semented ne$trophils ;;F
1ymphocytes );F
Monocytes 10F
<latelet co$nt ?;0.0000mm)
Erythrocyte sedimentation rate 7; mm0h
Ser$m
NaN 1)* mEC01
+lA 101 mEC01
KN ).2 mEC01
(+=)A 2) mEC01
9rea nitroen ,B9N- % m0d1
6l$cose 7) m0d1
+reatinine 0.2 m0d1
A !ari$m enema shows patches o# $lcerations alon the pro'imal colon with re#l$' o# dye into the
terminal ile$m. 3hich o# the #ollowin is the most appropriate ne't step in manaement4
A
- Eliminate #r$it J$ice #rom the patient7s diet
B
- Switch to a l$ten-#ree diet
+
- (2- receptor !loc2in aent therapy
8
- <rednisone and aminosalicylate therapy
E
- Bowel resection
2;. An asymptomatic ;7-year-old man comes to the physician #or a ro$tine health maintenance
e'amination. (e has smo2ed one pac2 o# ciarettes daily #or )7 years. (is !lood press$re is
1%00112 mm (. and p$lse is %20min. A!dominal e'amination shows a !r$it in the riht $pper
C$adrant and no masses. (is hematocrit is /2F. ser$m $rea nitroen ,B9N- le"el is 2) m0d1. and
ser$m creatinine le"el is 1./ m0d1. 3hich o# the #ollowin is the most li2ely ca$se o# this patient7s
!r$it4
A
- Acc$m$lation o# lipids in the arterial wall
B
- (ypertrophy o# the arterial wall media
+
- Bn#iltration o# arterial wall !y iant cells
8
- Bn#iltration o# ro$nd cells in the arterial wall
E
- >e#le' "asodilation
2?. An otherwise healthy /-month-old irl is !ro$ht to the physician !eca$se o# a !irthmar2 on her
arm that has increased in siDe o"er the past se"eral wee2s. E'amination shows a ) ' 2-cm. !riht
red. raised. so#t. nontender. compressi!le patch o"er the le#t #orearm. 3hich o# the #ollowin is the
most appropriate ne't step in manaement4
A
- :opical corticosteroid therapy
B
- Biopsy
+
- 1aser therapy
8
- E'cision
E
- No inter"ention is necessary
27. A 7-year-old irl is !ro$ht to the physician in Septem!er !eca$se o# #e"er and sore throat #or 1
day. She is in the third wee2 o# second rade. (er temperat$re is )%.? + ,101.; F-. E'amination
shows an erythemato$s pharyn' and slihtly enlared tonsils witho$t e'$date. :here is no
sini#icant cer"ical lymphadenopathy. A rapid test #or ro$p A streptococc$s is neati"e. 3hich o#
the #ollowin is the most appropriate ne't step in manaement4
A
- Monospot test
B
- :hroat c$lt$re
+
- Bntram$sc$lar penicillin therapy
8
- =ral erythromycin therapy
E
- =ral penicillin therapy
:he response options #or the ne't two items are the same. Io$ will !e reC$ired to select one answer
#or each item in the set.
For each patient with wea2ness. select the most appropriate test to esta!lish the ca$se o# the
condition.
A
- Brain stem a$ditory e"o2ed potentials
B
- +arotid $ltrasonoraphy
+
- +hromosomal analysis #or trin$cleotide repeat
8
- Electroencephaloraphy
E
- Electromyoraphy and ner"e cond$ction st$dies
F
- M>B o# the spine
6
- M$scle !iopsy
(
- >epetiti"e ner"e stim$lation
B
- Somatosensory e"o2ed potentials
H
- S<E+: scan
K
- :emporal artery !iopsy
1
- Gis$al e"o2ed potentials
2%. A ;7-year-old man is !ro$ht to the emerency department ? ho$rs a#ter the onset o#
wea2ness o# his riht #ace. arm. and le. :hree days ao. he had an episode o# s$dden "is$al loss in
his le#t eye that he descri!es as Ra shade comin down.R :he episode resol"ed completely within 10
min$tes. (e has hypertension and type 2 dia!etes mellit$s. !oth poorly controlled with lisinopril and
ly!$ride. E'amination shows e'pressi"e aphasia and riht lower #acial droop. :here is moderate
wea2ness on the riht. worse in the $pper e'tremity than the lower e'tremity. 8eep tendon re#le'es
are )N in the riht e'tremities and 2N in the le#t e'tremities. Ba!ins2i7s sin is present on the riht.
Sensory e'amination shows no a!normalities.
For each patient with wea2ness. select the most appropriate test to esta!lish the ca$se o# the
condition.
A
- Brain stem a$ditory e"o2ed potentials
B
- +arotid $ltrasonoraphy
+
- +hromosomal analysis #or trin$cleotide repeat
8
- Electroencephaloraphy
E
- Electromyoraphy and ner"e cond$ction st$dies
F
- M>B o# the spine
6
- M$scle !iopsy
(
- >epetiti"e ner"e stim$lation
B
- Somatosensory e"o2ed potentials
H
- S<E+: scan
K
- :emporal artery !iopsy
1
- Gis$al e"o2ed potentials
2*. A pre"io$sly healthy 27-year-old woman is !ro$ht to the emerency department !eca$se o# a
2-day history o# wea2ness o# her arms and les and n$m!ness o# her hands and #eet and a /-ho$r
history o# mild shortness o# !reath while s$pine. :he wea2ness !ean in her #eet and has
proressed to in"ol"e the hands. She descri!es a sensation o# Relectrical shoc2sR e'tendin #rom the
!$ttoc2s to the #eet. :oday. she tripped and #ell se"eral times and was $na!le to !$tton clothes or
hold $tensils. :hree wee2s ao. she had a mild $pper astrointestinal illness that resol"ed within ;
days. (er respirations are 200min and shallow. :here is mild #acial wea2ness& cranial ner"es are
otherwise intact. M$scle strenth in the $pper and lower e'tremities is /0; pro'imally and 20;
distally. :here is are#le'ia. Ba!ins2i7s sin is a!sent !ilaterally. Sensation to "i!ration is slihtly
decreased at the #iners and toes.
)0.
A 72-year-old man comes to the physician with his wi#e !eca$se o# chronic a!dominal pain and
headaches #or / months. (is wi#e states that he has !ecome more #oret#$l o"er the past ? months.
(e has a history o# o$t. (e has smo2ed one pac2 o# ciarettes daily #or ;0 years and drin2s 10 oD
o# homemade whis2ey daily. (e ta2es no medications. (is temperat$re is )?.% + ,*%.) F-. !lood
press$re is 1?00*% mm (. p$lse is 7/0min. and respirations are 1?0min. Ne$roloic e'amination
shows mild short-term memory loss and decreased sensation to pinpric2 in the distal e'tremities.
(e has an ata'ic ait. :here are o$ty tophi on the dorsal aspect o# the le#t el!ow. 1a!oratory
st$dies show:
(ematocrit ))F
Mean corp$sc$lar "ol$me 70 Em)
Ser$m
9rea nitroen ,B9N- 17 m0d1
6l$cose *0 m0d1
+reatinine 2 m0d1
9ric acid 1/ m0d1
3hich o# the #ollowin is the most appropriate ne't step in manaement4
A
- Meas$rement o# !lood lead le"el
B
- Meas$rement o# ser$m porpho!ilinoen le"el
+
- +: scan o# the a!domen
8
- M>B o# the !rain
E
- +ar!idopa-le"odopa therapy
)1. A co$nty health o##icer in"estiates an o$t!rea2 o# illness amon persons attendin a ch$rch
picnic. :he illness is characteriDed !y the onset o# na$sea and "omitin ) to / ho$rs a#ter attendin
the picnic. All a##ected persons reco"er witho$t speci#ic therapy. :he in"estiation implicates e
salad as the "ehicle o# transmission. :his episode is consistent with a #ood!orne o$t!rea2 ca$sed !y
which o# the #ollowin4
A
- +lostridi$m per#rinens
B
- 6iardia lam!lia
+
- Salmonella species
8
- Staphylococc$s a$re$s
)2. A ?2-year-old woman comes to the physician !eca$se o# eneraliDed wea2ness #or 2 wee2s. She
has a 20-year history o# arthritis o# the hands treated with aspirin and acetaminophen. She had two
episodes o# $rinary tract in#ections ; and 11 years ao. respecti"ely. E'amination shows no
a!normalities e'cept #or (e!erden7s nodes on the hands. 1a!oratory st$dies show:
Erythrocyte sedimentation rate
1; mm0h
Ser$m
NaN
1)? mEC01
+lA
100 mEC01
KN
/.* mEC01
(+=)A
20 mEC01
9rea nitroen ,B9N-
/1 m0d1
+reatinine
/ m0d1
9rine
<rotein
1N
3B+
2A/0hp#
>B+
none
Bacteria
none
SC$amo$s epithelial cells
occasional
6ran$lar casts
occasional
>enal $ltrasonoraphy shows no a!normalities. 3hich o# the #ollowin is most li2ely to ha"e
pre"ented this condition4
A
- <eriodic <<8 s2in testin
B
- Screenin #or a$toimm$ne ca$ses o# lomer$lonephritis
+
- <eriodic renal $ltrasonoraphy
8
- A"oidance o# analesics
E
- S$ppressi"e anti!iotic therapy #or treatment o# $rinary tract in#ections
)). A ?0-year-old man comes to the physician !eca$se o# increasin shortness o# !reath #or 1
wee2& it occ$rs at rest and is e'acer!ated !y e'ertion. (e has not had chest pain. (e has smo2ed
two pac2s o# ciarettes daily #or /0 years. Between the aes o# 1% and )0 years. he wor2ed in a
wareho$se with e'posed !are ins$lation& #or the past 1; years. he has dri"en a ta'i. Medications
incl$de ipratropi$m !romide and al!$terol metered-dose inhalers. (is temperat$re is )7 + ,*%.? F-.
!lood press$re is 1700*; mm (. p$lse is 1000min. and respirations are 2/0min. Anterior and
posterior di##$se wheeDes are heard. +ardiac e'amination shows no m$rm$rs. :here is mild preti!ial
edema. An '-ray #ilm o# the chest shows calci#ied ple$ral plaC$es on the riht hemidiaphram and a
2-cm ple$ral-!ased mass. 3ith reard to the #indins on the '-ray #ilm. which o# the #ollowin is
most li2ely to ha"e pre"ented this patient7s condition4
A
- Appropriate imm$niDations
B
- 8i##erent occ$pation
+
- 8i##erent medications
8
- (ypertension screenin
E
- Smo2in cessation
)/. A pre"io$sly healthy ?7-year-old man is admitted to the hospital !eca$se o# lethary. con#$sion.
m$scle cramps. and decreased appetite #or 7 days. (e appears ill. (is temperat$re is )7 + ,*%.? F-.
!lood press$re is 120070 mm (. p$lse is *%0min. and respirations are 200min. Breath so$nds are
diminished at the riht l$n !ase. Ne$roloic e'amination shows no a!normalities e'cept #or
lethary. Ser$m st$dies show:
NaN
11/ mEC01
KN
/.) mEC01
+reatinine
1 m0d1
:hyroid-stim$latin hormone
/.1 E90m1
An '-ray #ilm o# the chest shows a 2-cm nod$le in the riht lower lo!e and mediastinal adenopathy.
A !iopsy specimen o# the nod$le is most li2ely to show which o# the #ollowin4
A
- Adenocarcinoma
B
- +lear cell carcinoma
+
- Mesothelioma
8
- Small cell carcinoma
E
- SC$amo$s cell carcinoma
);. A ?2-year-old woman comes to the physician !eca$se o# !loatin and crampin a!dominal pain
and intermittent diarrhea o"er the past ; years. (er symptoms ha"e increased o"er the past month
since she started a new diet that emphasiDes yo$rt and cottae cheese as low-#at so$rces o#
calci$m and protein. Gital sins are within normal limits. A!dominal e'amination shows di##$se
tenderness to palpation with no re!o$nd tenderness& there are no masses or oranomealy. Bowel
so$nds are increased. :est o# the stool #or occ$lt !lood is neati"e. 3hich o# the #ollowin is the
!est e'planation #or this patient7s diarrhea4
A
- Bmpaired intestinal motility
B
- Bn#lammatory process
+
- Mala!sorption
8
- Secretory process
)?. A ;2-year-old woman comes to the physician !eca$se o# a )-month history o# intermittent
!loody dischare #rom the riht !reast. She does not per#orm re$lar monthly !reast sel#-
e'aminations. She has a )-year history o# maJor depressi"e disorder treated with #l$o'etine.
E'amination o# the !reasts shows no a!normalities. No masses are noted on palpation.
Serosan$ineo$s #l$id can !e e'pressed #rom the nipple o# the riht !reast !y pressin on the le#t
side o# the areola. 3hich o# the #ollowin is the most li2ely dianosis4
A
- +ystosarcoma phyllodes
B
- Fat necrosis
+
- Fi!roadenoma
8
- Fi!rocystic chanes o# the !reast
E
- 6alactorrhea
F
- (yperprolactinemia
6
- Bntrad$ctal papilloma
(
- Mastitis
B
- <aet7s disease o# the !reast
)7. A ?-month-old irl is !ro$ht to the physician !eca$se o# poor #eedin and la!ored !reathin #or
2 months. She has had rec$rrent respiratory tract in#ections since !irth. E'amination shows a to-
and-#ro m$rm$r in the second le#t intercostal space. a lo$d S2. !o$ndin peripheral p$lses. and a
widened p$lse press$re. 3hich o# the #ollowin is the most li2ely dianosis4
A
- Atrial septal de#ect ,osti$m prim$m type-
B
- Atrial septal de#ect ,osti$m sec$nd$m type-
+
- Atrio"entric$lar canal
8
- +oarctation o# the aorta
E
- (ypoplastic le#t heart syndrome
F
- <atent d$ct$s arterios$s
6
- :etraloy o# Fallot
(
- :ransposition o# the reat arteries
B
- :ric$spid atresia
H
- Gentric$lar septal de#ect
)%. A 70-year-old man comes to the physician !eca$se o# $rinary hesitancy and #reC$ency #or *
months. (is temperat$re is )7.; + ,**.; F-. E'amination shows a circ$mcised penis with no
$rethral dischare. :estic$lar e'amination shows no a!normalities. >ectal e'amination shows an
enlared r$!!ery prostate that is nontender to palpation. 9rinalysis shows many le$2ocytes and no
erythrocytes. 6ram7s stain o# $rine shows ram-neati"e rods. 3hich o# the #ollowin is the most
li2ely ca$se o# this patient7s condition4
A
- Bn#ection o# the epididymis
B
- Bn#ection o# the prostate
+
- Bn#ection o# the $rethra
8
- Ne$roenic !ladder
E
- =$t#low o!str$ction o# the !ladder
)*. A )2-year-old woman comes to the physician !eca$se o# lethary and !oredom since the !irth
o# her son ; months ao. She worries a!o$t her a!ility to care #or him and has had #reC$ent
palpitations. She is $na!le to #all !ac2 asleep a#ter nihttime #eedins. She stopped !reast-#eedin 1
month ao. (er son is healthy. and rowth and de"elopment are normal #or his ae. (er !lood
press$re is 1220%0 mm (. p$lse is ;%0min. and respirations are 1%0min. <hysical e'amination
shows no a!normalities. She remem!ers one o$t o# three o!Jects a#ter ; min$tes. (er ser$m
cholesterol le"el is 2?; m0d1. :he most appropriate ne't step in dianosis is meas$rement o#
which o# the #ollowin ser$m le"els4
A
- +ortisol
B
- Estroen
+
- <roesterone
8
- <rolactin
E
- :hyroid-stim$latin hormone
/0. :he enetic disease instit$te at a $ni"ersity hospital has de"eloped a rapid screenin test #or a
serio$s !$t treata!le inherited meta!olic disorder. Altho$h this disorder is predominantly #o$nd in
a partic$lar ethnic ro$p. it is also #o$nd sporadically thro$ho$t the entire pop$lation. :he
screenin test has a sensiti"ity o# *;F and a speci#icity o# *0F. 3hen $sed in an ethnically
prescreened pop$lation where the pre"alence o# this disorder is )0F. the positi"e predicti"e "al$e is
%0F and the neati"e predicti"e "al$e is *?F. :he instit$te proposes to $se this screenin test on
the eneral pop$lation where the pre"alence o# this disease is 0.1F. 3hich o# the #ollowin is the
most li2ely res$lt o# this screenin proram4
A
- Neati"e predicti"e "al$e decreases
B
- <ositi"e predicti"e "al$e decreases
+
- Sensiti"ity decreases
8
- Speci#icity decreases
/1. A )2-year-old woman comes to the physician !eca$se o# a 1-year history o# increasinly se"ere
d$ll pain in her lower !ac2 and !$ttoc2s. She also has had mornin sti##ness o# the lower !ac2 that
lasts #or 1 to 2 ho$rs. :here is no history o# tra$ma. (er Jo! does not reC$ire hea"y li#tin. (er
!lood press$re is 110070 mm (. p$lse is ?%0min and re$lar. and respirations are 1?0min.
+ardiop$lmonary e'amination shows no a!normalities. :here is tenderness to palpation o"er the
sacroiliac Joints !ilaterally and decreased #le'ion and e'tension o# the l$m!ar spine. An '-ray #ilm o#
the l$m!osacral spine shows sclerosis o# the sacroiliac Joints. 1a!oratory st$dies are most li2ely to
show which o# the #ollowin #indins in this patient4
A
- 8ecreased erythrocyte sedimentation rate
B
- (istocompati!ility h$man le$2ocyte antien B27
+
- Bncreased ser$m antin$clear anti!ody titer
8
- Bncreased ser$m carcinoem!ryonic antien ,+EA- le"el
E
- <ositi"e ser$m rhe$matoid #actor
/2. A ?-month-old !oy is !ro$ht to the physician !eca$se o# respiratory distress #or 1 day. (e had
a persistent dry co$h ) days ao. (e has not had a #e"er or nasal dischare. !$t he has had
chronic watery stools. (e is at the ;0th percentile #or heiht and 10th percentile #or weiht. (e
appears ill and is in respiratory distress. (is temperat$re is )7 + ,*%.? F-. p$lse is 1/00min. and
respirations are 7%0min. <$lse o'imetry shows an o'yen sat$ration o# 70F while !reathin room
air. E'amination shows white plaC$es on the m$co$s mem!ranes o# his mo$th and di##$se
adenopathy. :here are intercostal retractions. and di##$se crac2les are heard thro$ho$t all l$n
#ields. Bn addition to o'yen and anti!iotic therapy. which o# the #ollowin is the most appropriate
ne't step in manaement4
A
- Stool c$lt$re #or !acterial pathoens
B
- 9rinalysis
+
- (BG testin
8
- O$antitati"e meas$rement o# imm$nolo!$lins
E
- Nitro!l$e tetraDoli$m testin
F
- <latelet morpholoy e"al$ation
6
- :ympanocentesis
/). A /2-year-old man is !ro$ht to the physician !y his wi#e !eca$se o# a 2-month history o#
starin spells that last 1 to 2 min$tes each. 8$rin episodes. he also smac2s his lips and pic2s at his
shirt collar. Fo$r years ao. he was comatose #or 2 wee2s a#ter s$stainin a head inJ$ry in a
motorcycle collision& he reC$ired ? months o# reha!ilitation. (e reports that. o"er the past year. he
has had intermittent episodes o# smellin !$rnt r$!!er that occ$r appro'imately e"ery 2 wee2s. (e
hears an intense hissin so$nd d$rin these episodes. E'amination shows no a!normalities. 3hich
o# the #ollowin is the most li2ely dianosis4
A
- A!sence seiD$res
B
- +omple' partial seiD$res
+
- :ransient ischemic attac2
8
- :o$rette7s disorder
E
- 1im!ic encephalopathy
//.
A /7-year-old woman is admitted to the hospital #or e"al$ation o# a 2-wee2 history o# increased
irrita!ility and racin tho$hts. (er #amily reports that she ar$es with anyone who does not aree
with her. She has !een tal2in incessantly at a rapid rate and pacin aro$nd the ho$se. She says
that. #or years. she has !een hearin the "oice o# 6od tellin her that she has !een chosen #or a
special mission. She has !arely slept #or the past wee2. waitin to hear #$rther messaes #rom 6od.
She is dishe"eled and dehydrated. Ser$m st$dies show:
NaN 1)7 mEC01
+lA *? mEC01
KN ).% mEC01
(+=)A 22 mEC01
9rea nitroen ,B9N- )0 m0d1
6l$cose ?% m0d1
+reatinine 1.2 m0d1
Mental stat$s e'amination shows disoraniDed tho$hts and #liht o# ideas. She is aitated and has
di##ic$lty stayin seated. She is too distracti!le to per#orm coniti"e tests. :o'icoloy screenin is
neati"e. 3hich o# the #ollowin is the most li2ely dianosis4
A
- Ac$te renal #ail$re
B
- Borderline personality disorder
+
- 8ia!etes insipid$s
8
- Narcissistic personality disorder
E
- SchiDoa##ecti"e disorder
F
- SchiDoid personality disorder
/;. A 20-year-old man comes #or a ro$tine health maintenance e'amination. (e has a 1;-year
history o# di##ic$lty rela'in his hands a#ter tihtly rippin o!Jects or a#ter sha2in hands. (is
#ather has cataracts and #rontal !aldness. E'amination shows thin #orearms. :here is moderate
wea2ness o# the hands and di##ic$lty releasin a#ter rippin with his hands. 3hich o# the #ollowin
is the most li2ely dianosis4
A
- Amyotrophic lateral sclerosis
B
- +er"ical spondylosis
+
- M$ltiple sclerosis
8
- Myasthenia ra"is
E
- Myasthenic ,1am!ert-Eaton- syndrome
F
- Myotonic m$sc$lar dystrophy
6
- <olymyalia rhe$matica
(
- <olymyositis
B
- <roressi"e ne$ropathic ,peroneal- m$sc$lar atrophy
/?. A /7-year-old man is admitted to the hospital a#ter threatenin to harm a radio anno$ncer he
!elie"ed was !roadcastin his tho$hts. ="er the past 20 years. he has had m$ltiple psychiatric
hospitaliDations #or threatenin people who he !elie"ed were plottin aainst him. tryin to control
his mind. or ca$sin him to hear "oices !y implantin de"ices in his head. <ast symptoms impro"ed
with ne$roleptic therapy& a#ter dischare. he discontin$ed the medication and his symptoms
worsened. 3hich o# the #ollowin is the most appropriate pharmacotherapy to decrease this
patient7s ris2 #or #$t$re hospitaliDation4
A
- +loDapine
B
- Fl$phenaDine hydrochloride
+
- (aloperidol decanoate
8
- >isperidone
E
- :ri#l$operaDine hydrochloride
PPPPPPPPPPPPPPPPPPPPPPPPPPPPPPPPPPPPPPPPPPPPPPPPPPPPPPPPPPPPPPPPPPP
Answers:
NBME +K #orm 1
!loc2 1
c!!!c #daJh cac# c!#ed !d!ed daa!! ecaa! de!ed cacea d
!loc2 2
ddadd e#ec ed!ec dahh !!ee! cdeaa ad!#e eddde dd!!! !
!loc2 )
#c!!e cacae ccced a!cd caacc caea! !!e!a da!aa dced!
!loc2 /
heddd #pda hhi!d dcec! aceda ea!ea ddedc #ee! !c!e# c
NBME F=>M 1 ANS3E>S
B1=+K 1
1- +
2 -B
) -B
/- B
; -+
? -F
7- 8
% -B
* -(
10 -
11-+ 0 B 4
12- A
1)- 6
1/ A+
1;- F
1?-+
17- B
1%- F
1*- E
20 A8
21- B
22-8 0 + 4
2)-B
2/-E
2;-8
2?-8
27-A
2%-A
2*-B
)0- B
)1- E 0 8
)2-+
))-+
)/-A
);-B
)?-8
)7-E
)%-B
)*-E
/0-8
/1-+
/2-A
/)-+
//-E
/;-A
/?-8
!loc2 2
1-!
2-d4
)-a
/-d
;-d
?-e
7-#
%-e 0 ne't 4 c
*-44
10-c
11-e
12-d
1)-B0 d44
1/-e
1;-c
1?- 8
17- 6
1%- E
1*- (
20- K
21- B
22- B
2)- E
2/- E
2;- E
2?- +
27- B
2%- E
2*- A
)0- A
)1- A
)2- + 0 8
))- B
)/- F
);- E
)?- E
)7- 8
)%- 8
)*- 8
/0- E
/1- 8
/2- 8 0 E
/)- B
//- B
/;- B
/?- B
B1=+K )
1 F
2 +
) B
/ B
; E
? +
7 A
% + => E , i# strept pyoens is the same as strept pne$monia then its E-
* A
10 E
11 +
12 +
1) A
1/ E
1; 8
1?.a
17.!44
1%.
1*.a4c4
20.!4d4
21.c
22.a
2).a
2/.c
2;.a4!4
2?.c
27.a4!4
2%.e44
2*.a
)0.!
)1 B
)2 B
)) E
)/ B
); A
)? 8
)7 A
)% B ,why s$ch a low !p in this patient.-4
)* E
/0 B
/1 8
/2 +
/) E
// 8
/; B
/? 6
B1=+K /
1-m
2-e
)-e
/-d
;-d
?-
7-#
%-C
*-a
10-a
11-h
12-4a
1)-c
1/-4!
1;-!
1?-4c
17-c
1%-e
1*-4c
20-!
21-a
22-c
2)-e
2/-d
2;-!
2?-e
27-c
2%-4!
2*-e
)0-a
)1-d
)2-d
)) e
)/-d
);-c
)?-
)7-#
)%-e
)*-e
/0-!
/1-!
/2-c
/)-e
//-e
/;-i
/?-a

Вам также может понравиться